Magnetism

You might also like

Download as pdf or txt
Download as pdf or txt
You are on page 1of 62

MAGNETISM Rg.

2017 - 2019

PREFACE

Have you ever wondered exactly how a magnet sticks to your fridge? .Here you will
learn what magnetic domains are and how they work as well as why a compass needle
always points to magnetic poles. Plus, you will discover what magnetic field is and how
you can create it using an electric current. Magnetism is an attractive topic so, jump right
in!
This booklet consists of summarized text coupled with sufficient number of solved
examples of varying difficulties, which enables the students to develop problem solving
ability along with emphasis on physical concept. The problems are categorized into five
section, namely Exercise – I (objectives where only one of the option is correct),
Exercise – II (objectives where more than one option may be correct), Exercise – III
(matrix matches and paragraph type questions), Exercise – IV (subjective questions),
Exercise- V (old JEE questions) to help the student assess his understanding of the
concept and further improvise on his problem solving skills. Solutions to all the
questions in the booklet are available and will be provided to the students (at the
discretion of the professor). Every possible attempt has been made to make the booklet
flawless. Any suggestions for the improvement of the booklet would be gratefully
accepted and acknowledged.

(Dept. of Physics)

IIT –ian’s PACE

CENTERS : MUMBAI / DELHI /AKOLA / LUCKNOW / NASHIK / PUNE / NAGPUR / BOKARO / DUBAI
MAGNETISM Rg. 2017 - 2019

MAGNETISM
1. INTRODUCTION:
The phenomenon of attracting & repelling magnetic substances (viz, iron,nickel, cobalt etc.) is called
magnetism. Materials, which show above property are called magnets. Magnets can be of different
types.

MAGNET

NATURAL MAGNET ARTIFICIAL MAGNET

BASED ON MATERIAL BASED ON CURRENT

1.1 Natural Magnet:


In nature, a typical black stone is found which has a property of attracting tiny iron particles towards
itself. It is iron oxide ( Fe3 O4 ) and is called natural magnet.
(a) Natural magnets have no definite shape and size.
(b) Natural magnets have low attractive power.
(c) If a natural magnet is tied at its centre of gravity and is suspended freely in a horizontal plane, it
always stays in north-south direction.

1.2 Artificial Magnet:


It is a material upon which magnetic properties are imposed artificially.
(a) It has definite shape and size (eg., bar, horse-shoe, needle etc.)
(b) It has very large attractive power which can be varied, depending upon the requirement.

2. MAGNETIC FIELD (B)


The space around a magnet or the space around a current carrying conductor, in which their magnetic
effects can be experienced, is called as magnetic induction field.
(a) It is also known as magnetic field intensity or magnetic induction or magnetic field strength.
(b) It is a vector quantity.
(c) Its units in SI is 1 Wb / m2  1 tesla
and in CGS. 1 maxwell/cm 2 =1 gauss ;where 10 4 gauss = 1 tesla

2.1 Magnetic Line Of Force:


This concept is introduced to visualize the effect of the magnetic field. It is purely an imaginary
concept which helps us to understand the phenomenon, actually no such lines exist.
(a) Magnetic field lines represent the magnetic field in the same way as the electric field lines
represent an electric field.
(b) In order to draw these lines, we have to take a test object which is a magnetic dipole (e.g.,
compass needle)
(c) The path along which the compass needles are aligned is known as magnetic lines of force.

(d) The tangent to a field line at any point gives the direction of magnetic field B at that point.

CENTERS : MUMBAI /DELHI /AKOLA /LUCKNOW /NASHIK /PUNE /NAGPUR /BOKARO /DUBAI # 1
MAGNETISM Rg. 2017 - 2019
2.2 Characteristics of Magnetic lines of force :
(a) Magnetic lines of force are closed continuous curve starting from
north pole and going to south pole of the magnet and within the
magnet they run from south to north.

(b) Lines of force are in a state of tension which causes them to shorten longitudinally.

(c) Lines of force in a magnetic field have a tendency to repel


each other laterally.

(d) If in any region in a combined field due to two magnets there


are no lines of force, i.e., resultant force is zero at that region.
Such regions are called null or neutral points.

(e) Lines of force in a field also represent qualitatively the strength of the field at a point in the field.
Lines of force crowd themselves where the field is intense and spread themselves out where the field
is weak.

(f) Lines of force do not intersect, other wise the field should have two direction at the point of
intersection which is impossible.
(g) Magnetic monopoles do not exit.

3. THE LORENTZ FORCE LAW (MOTION OF CHARGED PARTICLE IN MAGNETIC


FIELD)
3.1 Force on moving charge:
The magnetic force on a charge q, moving with velocity v in a magnetic field B is given by
  
Fmag  q(v  B )
This is known as the Lorentz force law. If electric and magnetic both fields are present the net force
   
on q would be F  q[E  (v  B)]

3.2 Magnitude of magnetic force:


 
F  q v B sin  (where q is without sign and  is the angle between v & B ).
z F
3.3 Direction:
Force is perpendicular to both velocity and magnetic field. Its 
    B y
direction is same as v  B if q is positive and opposite of v  B if q is o
90

negative. 90o

Cases: v
x
(i) If q = 0, then F = 0 i.e., neutral particles do not experience
magnetic force.
(ii) If v = 0 , then F = 0 i.e., stationary charges do not experience magnetic force.
(iii) If  = 0 or  , then F = 0 i.e., when a charge moves parallel or anti-parallel to magnetic field it
does not experience magnetic force.
(iv) If  =  /2 then F = qvB is maximum. If B is uniform the particle will trace a circular path, if r
is the radius of the path then for circular motion
mv 2 mv p 2 mE
 qvB  r  
r qB qB qB

CENTERS : MUMBAI /DELHI /AKOLA /LUCKNOW /NASHIK /PUNE /NAGPUR /BOKARO /DUBAI # 2
MAGNETISM Rg. 2017 - 2019
Time period of circular motion:
2r 2m
T  T
v qB

Note: If   90 and B is not uniform then the path will not be circular.

Illustration 1: A proton and an  -particle is projected with same velocity in same magnetic field such that
 
v is perpendicular to B then find the ratio in their orbital radius.
R m q m 2e 1
Solution: Ratio in their orbital radius. p  p  a  p   R p : R  1 : 2
Ra ma q p 4 m p e 2

Illustration 2: A proton and a deuteron enter into a magnetic field B with same momentum which is
perpendicular to field. Find the ratio of time period of their circular motion.
p
Solution: We know, T  m , or T  as, p p  pd and md  2m p
v
Tp vd 1
 v p  2 vd   
Td v p 2
 
3.4 Motion of a charged particle when v and B are not perpendicular
Magnetic force on the particle F  qvB sin  y
If the field is uniform the particle move along helix.
Velocity of particle can be divided into two components one parallel
   v cos 
to B and other perpendicular to B In parallel direction of B the particle will 
B
x
v sin 
move with uniform velocity as it will not experience any magnetic force v
along this direction whereas due to the perpendicular component of
velocity it will undergo a circular path and these two motions together
z

mv sin 
gives a helical motion. Radius of helix, r 
qB
Distance between two successive identical parts of the helix, pitch
2m
p (v cos )
qB
2m
Time period of rotation, T 
qB
Note:
(1) Work done by magnetic force on moving charge is always zero.
(2) If a charged particle moves only in magnetic field then its kinetic energy remain unchanged.

Illustration 3: An infinite long wire of radius ‘a’ is carrying current I. An electron leaves the wire with
velocity v perpendicular to its length. Find the maximum displacement of electron from the
wire.
Solution: As the charge is moving in a magnetic field so work done v0
(0, 0) y
is zero i.e., speed will remain constant, a x x, y
y
2 2 Vy
Thus, v0  vx  v y Suppose at time t the charge is at (x, y)
V0
 Vx
So that v  v x ˆi  v y ˆj rmax
 0i ˆ
And its distance from axis is y.  B k
2y x
Thus, force on the charge

CENTERS : MUMBAI /DELHI /AKOLA /LUCKNOW /NASHIK /PUNE /NAGPUR /BOKARO /DUBAI # 3
MAGNETISM Rg. 2017 - 2019
   I
F  ( e)[v  B]  (e)[vxiˆ  v y ˆj ]  ( 0 kˆ)]
2y
 Ie dv
 Fx  0 v y  max  m x ….. (1)
2y dt
 Ie dv
and Fy   0 vx  ma y  m y ….. (2)
2y dt
r
v0 max
0 Ie dy dv  Ie dy  eI dy

2y dt
 m x  dv x  0 
dt 2m y
 0 dvx  20 m 
a
y
0 Ie rmax  2mv0 
 v0  n  rmax  a exp 
2 m a  0 Ie 

MULTIPLE CHOICE QUESTIONS


1 Suppose a uniform electric field and a uniform magnetic field exists along mutually perpendicular
direction in a gravity free space. If charge particle is released from rest, at a point in the space,
(a) Particle cannot remain in static equilibrium.
(b) Particle will move along a curve path but after sometime its velocity will become constant.
(c) Particle will come to rest at regular interval of time.
(d) Acceleration of the particle will never become equal to zero.

2 If a moving charged particle enters perpendicularly into a region of magnetic field from outside,
(a) It does not complete circular path. (b)It may complete circular path.
(c) It follows a straight line path. (d)It may follow a straight line path.

3 If a moving charged particle traces a helical path in a uniform magnetic field,


(a) Axis of the helix is parallel to the magnetic field.
(b)Axis of the helix is perpendicular to the magnetic field.
(c) Inclination of the axis of the helix with the magnetic field depends upon angle between direction
of motion of the particle and magnetic field.
(d) None.

4 Power associated with the force exerted by a magnetic field on a moving charged particle,
(a) Is always equal to zero.
(b) Is equal to zero only when direction of motion is perpendicular to magnetic field.
(c) Is equal to zero only when direction of motion is parallel to magnetic field.
(d) It never equal to zero.

5 If in a region an uniform magnetic field and uniform electric field both exist, a charge particle moving
in this region,
(a) Cannot trace a circular path. (b) May trace a circular path.
(c) May trace a straight line path. (d) Cannot move in the region with constant velocity

6 Two singly ionised isotopes of an element are accelerated from rest through the same potential
difference and they enter perpendicularly into an uniform magnetic field then,
(a) Their respective kinetic energy before entering into the magnetic field is equal.
(b) Their respective kinetic energy remains constant during motion in the magnetic field.
(c) They cannot trace a straight line path.
(d) They trace circular arcs in the magnetic field.

CENTERS : MUMBAI /DELHI /AKOLA /LUCKNOW /NASHIK /PUNE /NAGPUR /BOKARO /DUBAI # 4
MAGNETISM Rg. 2017 - 2019
7 In the previous objective:
(a) Since both the isotopes are accelerated through same potential difference, therefore, they trace
circular arcs of equal radii.
(b)Angular velocity of heavier isotopes will be greater than that of lighter isotopes.
(c) The radius of the circular arc traced by the heavier isotope in the magnetic field is greater than
that of the lighter isotope.
(d) The ratio of radii of the circular arcs traced in the magnetic field is equal to ratio of the magnitude
of momenta of two isotopes.

8 A charge particle moves in a gravity free space where an electric field of strength E and a magnetic
field of induction B exist. Which of the following statements is\are correct?
(a) If E is not equal to zero and B is also not equal to zero, velocity of the particle may remain
constant.
(b) If E is not equal to zero, particle cannot trace a circular path.
(c) If E is equal to zero, kinetic energy of the particle remains constant.
(d) None.

9 suppose a particle having small mass but strong electric charge is dropped from rest at top of a high
tower,
(a)Particle will follow a vertical straight line path.
(b)Particle may come to an instantaneous rest before striking to the ground.
(c)Particle may come into contact with the ground with velocity directed horizontally
(d)None

10 A charged particle enters into a space where a uniform gravitational field exists vertically
downwards and continues to move undeflected. Then in that space,
(a) A uniform horizontal electric field and a vertical magnetic field may be present.
(b) A vertical electric field alone may be present.
(c) Uniform electric and magnetic fields, both directed vertically downwards, may be present.
(d) A uniform horizontal magnetic field alone may be present.

ANSWERS KEY

1. (a,c,d) 2. (a) 3. (a) 4. (a) 5. (a,c)

6. (a,b,c,d) 7. (c,d) 8. (a,c) 9. (b,c) 10. (b,c,d)

4. MAGNETIC FORCE ON A CURRENT-CARRYING CONDUCTOR:


  
The equation F  q (v  B ) can be used to find the force on a current carrying conductor placed
inside the magnetic field B. Consider an element of length dl of an infinitely long wire carrying a
current i placed inside a field B as shown in the figure.

The free electrons in the wire drift with a speed vd known as the drift speed of the electrons in the
direction opposite to the current. The relation between the current i and the drift speed vd is
i = jA = nevd A

CENTERS : MUMBAI /DELHI /AKOLA /LUCKNOW /NASHIK /PUNE /NAGPUR /BOKARO /DUBAI # 5
MAGNETISM Rg. 2017 - 2019
Here A is the area of cross section and n is the number of free electrons per unit volume. The number
of electrons in an elemental volume Adl is nAdl. Thus the magnetic force on a wire of length dl will
be
  
dF  (nAdl)( ev d  B)
where q = -e is the charge on electron
If we denote length dl along the direction of the current by dl , the above equation becomes
  
dF  nAevd d l  B
Using (i),
  
dF  id l  B …………. (4.1)

The quantity id l is called a current element.

If a straight wire of length l carrying a current i is placed in a uniform magnetic field B , the force
acting on it is given by
  
F  il B …………. (4.2)
Points to remember regarding expression (4.2)
  
(1) Magnitude of F is , F = ilB sin θ, here θ is the angle between l and B . F is zero for
θ = 00. or 180 0 and maximum for θ = 900

(2) Here l is the vector that points in the direction of current i and has magnitude equal to the
length.
(3) The above expression applies only to a straight segment of wire in a uniform magnetic field.
(4) For the magnetic force on an arbitrarily shaped wire segment, let us consider the magnetic force

exerted on a small segment of vector length d l .
  
dF  i(d l  B) ………… (4.3)

Fig 4.1(a) Fig 4.1(b)


To calculate the total force F acting on the wire shown in the figure above, we integrate Eqn. 4.3 over
the length of the wire.
D  
 
F  i d l  B
A
………… (4.4)

Two special cases arise for the Eqn. 4.4


Case 1. A curved wire ACD as shown in the Fig. (a) Carries a current I and is located in a uniform
 
magnetic field B . Because the field is uniform, we can take B outside the integral in Eqn. 4.4 and
we obtain
  D  
F   i  d l   Bo …………. (4.5)
 A 
D 
But the quantity  dl represents the vector sum of all length elements from A to D. From the
A

polygon law of vector addition the sum equals the vector l directed from A to D. Thus,
  
F  il B …………..(4.6)
   
Or we can write, FACD  FAD  i ( AD  B)

CENTERS : MUMBAI /DELHI /AKOLA /LUCKNOW /NASHIK /PUNE /NAGPUR /BOKARO /DUBAI # 6
MAGNETISM Rg. 2017 - 2019
Case 2. An arbitrarily shaped closed loop carrying a current I is placed in uniform magnetic field as
shown in Fig. (b). We can again express the force acting on the loop in the form of Eqn.4.5, but this

time we must take the vector sum of the length element d lo
Over the entire loop,
  
 
F  i  d l  B
Because the set of length elements forms a closed polygon, the vector sum must be zero.

F 0
Thus, the net magnetic force acting on any closed current loop in a uniform magnetic field is zero.
(5) The direction of F can be given by Fleming’s left hand rule. According to this rule, the
forefinger, the middle finger and the thumb of the left hand are stretched in such a way that they are

mutually perpendicular to each other. If middle finger shows the direction of current (or l ) and

forefinger shows the direction of magnetic field ( B ), then the thumb will give the direction of
magnetic force (F).

Illustration 4: A triangular loop PQR carrying a current I. The triangle is equilateral with edge-length l . A
uniform magnetic field B exists in the direction parallel to PQ. Find the force acting on three
wires PQ, QR and RP separately.
Solution: The force on wire PQ is
    
F1  iPQ  B  0 as the field B is parallel to PQ
The force on QR is
    3
F2  iQR  B F2  ilBsin120  ilB
2
From the rule of vector product, this force is perpendicular to the plane of the diagram and is going
into it.
The force on RP is;
    3
F3  iRP  B F3  ilBsin120  ilB
2
From the rule of vector product, this force is perpendicular to the plane of the diagram and is coming
out of it.

5. TORQUE ON A CURRENT CARRYING LOOP


Consider a rectangular current carrying loop PQRS carrying a current i as shown in the figure below.
The magnetic forces F1, F2, F3 and F4 on the wires PQ, QR, RS and SP respectively are obtained
  
from equation F  i l  B . These forces act from the middle point T, U, V, W of the respective sides.
Hence F1 = F3 = ilB and F2 = F4 = ibB . The resultant force is therefore zero.
Also, F1 and F3 have the same line of action so they together produce no torque. Similarly F2 and F4
produce no torque.

CENTERS : MUMBAI /DELHI /AKOLA /LUCKNOW /NASHIK /PUNE /NAGPUR /BOKARO /DUBAI # 7
MAGNETISM Rg. 2017 - 2019
Now suppose the loop is rotated through an angle θ about the line WU. The wire PQ shifts parallel to
  
itself so that the force F1  i l  B on it remains unchanged in magnitude and direction. Its point of
application T shifts to T'. Similarly, the force on RS remains F3, But the point of application shifts to
V'. The line TV gets rotated by an angle θ to take the position T' V'. This line makes an angle θ with
the force F1 and F3. The torque of F1 about O has magnitude
   b  b
| OT '  F1 |    F1  sin   (ilB )sin 
2 2
b
This torque acts along the line UW. The torque of F3 about O is also (ilB ) sin 
2
Along the same direction. As the wire QR rotates about WU, the plane containing the wire and the
magnetic field does not change. The force on this wire is perpendicular to this plane and hence its
direction remains unchanged. Also, the point of application U remains the same. Similar in the case
for the wire SP. The forces on QR and SP are, therefore, equal and opposite and act along the same
line. They together produce no torque.
The net torque acting on the loop is therefore,
b b
Γ= (ilB) sin  + (ilB ) sin  = b(ilB ) sin 
2 2
= iAB sin 

Where A is the magnitude of area of the loop PQRS. The direction of the area vector A
Is perpendicular to the plane of the loop and is towards the side from which the current looks anti-
clockwise. Thus in the figure the area vector points towards a viewer. It is drawn from the center O

of the loop. Another way to get the direction of A is to use the right hand thumb rule. If you curl

your fingers of the right hand along the current, the stretched thumb gives the direction of A .
 
In the figure (a) the angle between the area vector A and the magnetic field B is zero. As the loop
rotates the area vector also rotates by an angle θ. Taking the direction of torque (along UW) into
consideration,
    
  iA  B    B ………. (5.1)
 
where   iA is called the magnetic dipole moment or simply magnetic moment of the current loop.
We know that in a electric dipole when the pair of charges +q and –q are separated by a distance l then the
electric dipole moment is given by p = ql and in the direction from –q to +q. If such a dipole is placed in a
  
uniform electric field, a torque   p  E acts on the dipole. Equation 5.1 is similar to this structure and

hence  is called the magnetic dipole moment. For n number of turns we write magnetic dipole moment as
 
  niA ……….. (5.2)
The equations 5.1 and 5.2 obtained are shown for a rectangular loop but are also valid for a plane loop of
any shape.
Every current carrying loop can be treated as a bar magnet. In a bar magnet, magnetic field lines
emanates from the north pole and after forming a closed path terminates on south pole. Similarly a current
carrying loop can be imagined to have a north pole and South Pole or in simple words it can be imagined
that a bar magnet is placed inside the loop. Thus current carrying loop forms a magnetic dipole. Now we
know that the direction of magnetic moment is from south pole to north pole and that the direction of
 
magnetic moment of the loop is in the direction of area vector A as in Eqn. 5.2. Since the direction of A is
decided by the stretched thumb as discussed above the stretched thumb will point in the direction of North
Pole.

CENTERS : MUMBAI /DELHI /AKOLA /LUCKNOW /NASHIK /PUNE /NAGPUR /BOKARO /DUBAI # 8
MAGNETISM Rg. 2017 - 2019
Illustration 5: Find the magnitude and direction of magnetic moment for the loops considered below

solution: In fig. (a) From the sense of current the direction of  into the
paper as indicated in the figure. Hence north pole is in the plane
of the paper and South Pole towards the side of the viewer.
Hence magnetic moment is perpendicular to paper inwards.
Magnitude of magnetic moment is

|  | = iπR2

Here opposite is the case. South Pole is into the paper and North
pole outside the paper. Therefore magnetic moment is
perpendicular to the plane of the paper in outward direction.
Magnitude of magnetic moment is

|  | = a2 i

In this case South Pole is on the right side of the loop and North

pole on the left side. Hence  is directed from right to left.
Magnitude of magnetic moment is

|  | = abi

6. FORCE BETWEEN PARALLEL CURRENT CARRYING WIRES

Consider two long wires 1 and 2 kept parallel to each other at a distance r and
carrying currents i1 and i2 respectively in the same direction.
Magnetic field on wire 2 due to current in 1 is given by Biot Savart’s law as
 i
B= 0 1 [in  direction]
2 r
Magnetic force on a small element dl of wire 2 due to this magnetic field is,
  
dF  i 2 (d l  B)

Magnitude of this force is, dF  i 2 [(dl)(B)sin 90 ]
 i   ii
 i2 (dl ) 0 1   0  1 2  dl
 2 r  2 r
 
Direction of this force is along d l  B or towards the wire 1.
The force per unit length on wire 2 due to wire 1 is,

dF  0 i1i2
  …………. (6.1)
dl 2 r
The same force acts on wire 1 due to wire 2. The wires attract each other if the current in the wires are
flowing in the same direction and they repel each other if the currents are in the opposite direction. The
force of attraction or repulsion between two current carrying wires is often called Ampere Force.

CENTERS : MUMBAI /DELHI /AKOLA /LUCKNOW /NASHIK /PUNE /NAGPUR /BOKARO /DUBAI # 9
MAGNETISM Rg. 2017 - 2019
MULTIPLE CHOICE QUESTIONS
1 An electron moving in a circular orbit around the nucleus of an atom,
(a) Exerts an electric force on the nucleus equal to that on it by the nucleus.
(b) Produces a magnetic induction at the nucleus.
(c) Has a magnetic dipole moment.
(d)Has a net energy inversely proportional to its distance from the nucleus.

2 A current carrying ring is place in a magnetic field. The direction of the field is perpendicular to the
plane of the ring
(a) There is no net force on the ring. (b)The ring may tend to expand.
(c) The ring may tend to contract. (d)None

3 A parallel beam of electrons is shot into an uniform electric field, initially parallel to and against the
field with a small initial speed. Then:
(a) The beam will pass through the field accelerating down the field without changing its width.
(b) The beam tends to spread out at the beginning and to narrow down later.
(c) The beam tends to narrow down at the beginning and to spread out later.
(d) The total energy of the beam is conserved.

4 Two long straight thin wires carrying constant currents are non co-planar and mutually
perpendicular. Which of the following statements is\are correct?
(a) They have tendency to move away from each other, parallel to their original positions.
(b) They have tendency to move towards each other, parallel to their original positions.
(c) They have tendency to rotate about the line of the shortest distance between them and approach
each other.
(d) They have tendency to rotate about the line of the shortest distance between them and move away
from each other.

5 A beam of electrons starts to accelerate from rest due to a uniform electric field in vacuum, the
moving electrons,
(a) Initially experience a force of mutual repulsion.
(b)Experience a force of mutual attraction after travelling a certain distance.
(c) Will follow parallel lines because there is no force of interaction between them.
(d) Will continue to diverge due to electrostatic repulsion.

6: A wire of given length is to be bend into a closed figure so that it may have the maximum magnetic
moment when it carries a certain current. It should be bend into,
(a) An ellipse. (b) A circle (c) A square. (d) Any shape.

ANSWERS KEY:

1(a,b,c,d) 2(a,b,c) 3(b,d) 4(c) 5(a,b) 6(b)

CENTERS : MUMBAI /DELHI /AKOLA /LUCKNOW /NASHIK /PUNE /NAGPUR /BOKARO /DUBAI # 10
MAGNETISM Rg. 2017 - 2019
7. THE BIOT-SAVART LAW
(a) This law is applicable to a small current carrying conductor.
(b) It is an experimental law which gives the quantitative value
of the magnetic field produced by a current element at any point
in the space around it.
(c) Let XY be a conductor of an arbitrary shape carrying current
‘i’ and P is a point in vacuum at which the field is to be
determined. We consider an infinitesimal current element AB of
length dl . Its direction is that of the tangent to the conductor. Let
r be the position vector of P making an angle  with dl .
(d) According to Biot-Savart’s Law, the magnitude of magnetic field dB (also called magnetic flux
density) at a point P due to current element dl of the current carrying conductor is
(i) Directly proportional to current, i.e., dB  i
(ii) Directly proportional to elemental length, i.e., dB  dl
(iii) Directly proportional to the sine of angle  between dl & r i.e., dB sin 
1
(iv) Inversely proportional to the square of distance dB 2
r
Hence,
idl sin   idl sin  
dB 2
or dB  0 2
Where 0  10 7 Wb / A  m
r 4 r 4
 0 is called permeability of free space and its value is 4 107 N / A 2

(e) Direction of d B . The direction of magnetic field is along the


direction of cross product dl  r i.e. vector dB is perpendicular to
both the current element idl and position vector r .

The direction of dB is represented by the


Right hand rule:
If the thumb represents the current and we curl our fingers to pass through the point P, then the
direction of the fingers at P gives the direction of magnetic field there. Remember, this law is
applicable for straight current carrying conductor. Combining all the above factors, we can write
 i (dl  r )
dB  0 …………….(7.1)
4 r3

(f) Superposition principle: Net magnetic field at a field point is equal to the vector sum of the
individual magnetic fields at that point, i.e., Bnet  B1  B2  .......
In accordance with the principle of superposition the total B is found as a result of integration of the
equation (7.1) over all current elements.

8. APPLICATION OF BIOT-SAVART LAW

8.1 Magnetic field on a point on the line of current: If a point lies on the line of current carrying
element, then magnetic field at this point is always zero.

B=0 I B=0
 
Q P

CENTERS : MUMBAI /DELHI /AKOLA /LUCKNOW /NASHIK /PUNE /NAGPUR /BOKARO /DUBAI # 11
MAGNETISM Rg. 2017 - 2019
8.2 Magnetic field due to a Straight current carrying Conductor
A
Suppose a straight current carrying conductor AB, carrying an
upward current I, is placed on the paper. P is a point at a
perpendicular distance R from conductor, where magnetic field is to
be determined. I 1
Rr   P
2
d

According to Biot-Savart’s law, the field at P due to element dl is dl

 0 Idl sin   I  dl sin   1


dB  2
 0 .  B
4 r 4  r  r
R  0 I cos
Now from the figure, dl sin   rd and , cos    dB  . d
r 4 R
Hence magnetic field due to the whole conductor

1 1
0 I 0 I 0 I
B  dB   cos d  [sin 1  sin  2 ] B  [sin 1  sin  2 ]
 2  2
4R 4R 4R
Different Cases

Case 1 : When the linear conductor XY Case 2 :Infinite long wire Case 3 : Semi infinite long wire
is of finite length and the point P lies on
it's perpendicular bisector as shown

 P

P r
i r i i
r
P
X
 1 =  2 = 90o. 1  90 and  2  0o .
o

1  2   0 i  2i 0 i  i
So, B  [sin90o  sin 90o ]  0 So, B  [sin 90o  sin 0 o ]  0
 i 4 r 4 r 4 r 4 r
So B  0 . (2 sin  )
4 r
Illustration 6: Find the magnetic field at the centre of a square loop of side ‘a’ carrying current I in
anticlockwise sense. D C

P
 I
Solution: BAB  0 (sin  / 4  sin  / 4)   /4 a/2
a
4
2 A B

Which is same for all the four wire,


0 I   2 20 I
 BP  4 . (sin  sin )  
a 4 4  a
4
2
Note:  means: directed out of the plane paper.

CENTERS : MUMBAI /DELHI /AKOLA /LUCKNOW /NASHIK /PUNE /NAGPUR /BOKARO /DUBAI # 12
MAGNETISM Rg. 2017 - 2019
Illustration 7: Find the net magnetic field at the origin of coordinates axis for the current carrying wires
ABC & DEF as shown
C
Solution: Here, BAB  BDE  0 y

 I
And BBC  BEF  0  a
a B I
4 D E O A
X
2 I
0 I
 Bnet  B  
a F

Illustration 8: Find the net magnetic field at point P on the perpendicular bisector of two current carrying
conductors A & B separated by a distance 2a as shown.

0 I
Solution: Here, due to A BA   BB
2 a 2  x 2
20 I a
Bnet  2 B sin   .
2 a 2  x 2 a 2  x 2
 0 Ia
 Bnet 
 (a 2  x 2 )
and is directed along horizontal. P

r
Illustration 9: An infinitely large sheet carries current with linear current density i.
Find the net magnetic field a point which is at a perpendicular distance r from
x
the sheet. dx

Solution: Let us consider a current carrying element-idx P


0
0 (idx)
 dB p  dB r 0
2 r 2  x 2

It has two components one parallel to the plane of the sheet and X
X dx
other perpendicular to it. dBx  dB cos and dB y  dB sin 
 
 0idx.r  0i  0idx.x i
 Bx   dBx   2 2
 and By   dBy   dBy  2 2
 0 B  0

2 (r  x ) 2 
2 (r  x ) 2

8.3 Magnetic field due to a current carrying Loop


(a) At an axial point and (b)At the center of the loop
(a) At an axial point:
Consider a circular loop of radius a carrying a current i. We
have to find the magnetic field at a point P on the axis of the
loop at a distance d from its center O. In the above figure the
loop is perpendicular to the plane of the figure while its axis is
in the plane of the figure. The current comes out of the plane at M and goes into it at N. Consider the
  
current element id l of the wire at M. The vector joining the element to the point P is r  MP . The
magnetic field at P due to this current element is

  i(d l  r)
0
dB 
4 r3

CENTERS : MUMBAI /DELHI /AKOLA /LUCKNOW /NASHIK /PUNE /NAGPUR /BOKARO /DUBAI # 13
MAGNETISM Rg. 2017 - 2019
  
As d l is perpendicular to the plane of the figure, d l  r must be in the plane. The figure shows the

direction of dB according to the rules of vector product. The magnitude of the field is

 0 idl  0i dl
dB  
4 r 2 4 a 2  d 2

The component along the axis is


 0ia dl
dB cos   ……….. (i)
4 ( a  d 2 ) 3 / 2
2

Now consider the diametrically opposite current element at N. The field due to this element will have
the same magnitude dB and its direction will be along the dotted arrow shown in the figure. The two
fields due to the elements at M and N have a resultant along the axis of the loop. Dividing the loop in
such pairs of diametrically opposite elements, we conclude that the resultant magnetic field at P must
be along the axis. The resultant field at P can, therefore, be obtained by integrating the right hand
side of (i), i.e.,
 0ia 1
B dl
4 ( a  d 2 ) 3 / 2
2

 ia 1 0 ia 2
 0  2 a B 
4 (a 2  d 2 )3 / 2 2 ( a 2  d 2 )3/ 2

The right-hand thumb rule can be used to find the direction of the field.
(b) At the center of the loop :
The magnetic field at the center of the loop is simply obtained by putting d = 0 in above Eqn., thus
obtaining
i
B 0
2a

Concentric circular loops (N = 1)


(i) Coplanar and concentric : It means both coils are in same plane with common centre
(a) Current in same direction (b) Current in opposite direction
i
i
i
i  1 1 r2 0 1 1 
r2 B1  0 2i    r1 B2  2i   
r1 4  r1 r2  4  r1 r2 

B1  r2  r1 
Note: (i)  
B2  r2  r1 
(ii) Non-coplanar and concentric: Plane of both coils are B2
perpendicular to each other i1 
Magnetic field at common centre
0 B1
B B12  B 22  i12  i 22  i2
2r

CENTERS : MUMBAI /DELHI /AKOLA /LUCKNOW /NASHIK /PUNE /NAGPUR /BOKARO /DUBAI # 14
MAGNETISM Rg. 2017 - 2019
8.4 Magnetic field due to a current carrying Arc
Arc making an angle  at the centre : To measure the magnetic field at the centre of a circular arc of
radius R carrying current- I making an angle  at the centre, we have to take a very small element at
angle  making d  at the centre.
Thus, I
 0 I Rd .sin 90  0 I Rd
dB  .  . d .
4 R2 4R d
  
0 I  0 I
 B d  R
4R 4R P
0

 I
or B 0 .
4 R
e.g.; Magnetic field at centre O
i i
i


r O r
O r
O 

 0 i  0i  i  (2   ) i
B .  B 0. B 0.
4 r 4r 4 r 4 r
  0 2i 
Magnetic field at the centre of circular coil is denoted by B0   . 
 4 r 

Illustration 10: Find the magnetic field at the centre of two coaxial semicircular arc of radius a & b carrying
a current I as shown.
Solution: I
BAB  BCD  0
 0 I
BBC  . 
2 2a b
 I a
or BBC  0 
4a
 0 I A B P C D
and BDA  . 
2 2b
 I 
or BDA  0
4b
 .I  1 1 
 Bnet    
4 a b
Note :  means : directed into the plane of paper.

8.5 Zero magnetic field : If in a symmetrical geometry, current enters from one end and exists from the
other, then magnetic field at the centre is zero.
i i

O
O
O

CENTERS : MUMBAI /DELHI /AKOLA /LUCKNOW /NASHIK /PUNE /NAGPUR /BOKARO /DUBAI # 15
MAGNETISM Rg. 2017 - 2019

O O O

In all cases at centre B  0


8.6 Magnetic field due to Solenoid
A solenoid is a long cylindrical helix which is obtained by winding closely a large number of turns
of insulated copper wire over a tube of card board or china clay. When electric current is passed
through it, a magnetic field is produced around and within the solenoid. If n be the number of turns
per unit length of the solenoid, then number of turns in width dx = ndx.
If ‘i’ current is passing through it then magnitude of magnetic induction at P.

 0 (ndx)iR 2
dB  along the axis
2( R 2  x 2 ) 3 / 2

2
1 R2  1 R2  2
2
 0 niR 2 dx
 Net magnetic induction B   dB  
 2
2( R 2  x 2 ) 3 / 2 1 2
P
 0 ni  1 2 
1 2
   
2  R2   2 2
R   2 
2
 1

 0 ni 0 ni
 cos1  cos  2  or B  cos  cos  
2 2


P

Which is directed along the axis.

Case I: For an ideal solenoid (i.e., solenoid of infinite length) R<< 


 Bin   0 ni (i.e. same everywhere) Bout  0
Case II: Semi-infinitely long solenoid (at one end)
 ni  ni
B  0 cos 0  cos  / 2  0
2 2

CENTERS : MUMBAI /DELHI /AKOLA /LUCKNOW /NASHIK /PUNE /NAGPUR /BOKARO /DUBAI # 16
MAGNETISM Rg. 2017 - 2019
9. BIOT-SAVART LAW FOR MOVING CHARGES
(a) Magnetic field at point P due to a moving charge q is given by
  0 q(v  r ) 
v P
B  [q is with sign]
4 r3

 r
(b) Thus, magnitude of magnetic field is
 qv sin  q
B 0 
4 r2
 
Where  is the angle between r & v and q is with sign

(c) Direction is determined by using Right hand rule for current corresponding to motion of charge.
v
i
q 
v
i
q 

9.1 Magnetic field at the centre of a circle:


(where a charge particle is orbiting along the circumference).
Consider a charge + q rotating in anticlockwise sense in a circle of
radius r with tangential speed v.
 qv
Thus, field at the centre, B  0 2 r
4r
 q 2 rf 0 2 fq -q
v
B 0  2

4 r 4 r

where f is the frequency of rotation.
i q
or B  0 where i  = qf , T is the period of rotation.
2r T
Note: Behavior of moving charge along the circle is equivalent to a current carrying conductor.

9.2 Magnetic field on the axis of a circle: As the charge is moving along the circle,
 qv sin  / 2
B 0  is the magnetic field at point P
4 ( R 2  x 2 )
+q
R2  x2
v B
R

x P

0 qv
Baxis   2 sin  When average is taken it behaves like current carrying ring.
4 ( R  x 2 )
R
where sin  
R  x2
2

CENTERS : MUMBAI /DELHI /AKOLA /LUCKNOW /NASHIK /PUNE /NAGPUR /BOKARO /DUBAI # 17
MAGNETISM Rg. 2017 - 2019
9.3 When two charges are placed symmetrically and moving with same speed:
0 qv
 Baxis   2. sin 
4 ( R 2  x 2 )
q

0 q 2 Rf R 0 qfR 2 v
  
4 ( R 2  x 2 )3/2 2 ( R 2  x 2 ) 3/2
 0iR 2

2( R 2  x 2 ) 3 / 2 q
v

9.4 Magnetic field due to rotating dipole:


+q
0 qvx  B- B+
Here, B  v
4 ( 2  x 2 )3/2
20 qvx
Bnet  
4 ( 2  x)3/2
-q v

Note:
(1) Behavior of moving charge along circle is not equivalent to current carrying coil for
instantaneous value of magnetic field on its axis.
(2) For average value of magnetic induction on the axis behaviour of moving charge is equivalent to
the current carrying coil. Where current = charge  frequency
(3) For symmetric charge distribution behaviour is equivalent to current carrying coil for both
instantaneous and average value.

Illustration 11: Find the magnetic field due to a rotating uniformly charged ring. For any point on axis or
centre
 qf q
Solution: Bcentre  0 and
2R
v
0 qfR 2 R
Baxis 
2( R 2  x 2 )3/2

I = qf

MULTIPLE CHOICE QUESTIONS

1. A vertical wire carries a current upwards. The magnetic field at a point due to north of the wire is
directed.
(a) Upward (b) due south (c) due west (d) due east

2. AB is a section of a straight wire carrying a current I. P is a point at a distance d


from AB. The magnetic field at P due to AB has magnitude
o I o I
(a) (cosθ 1 +cosθ 2 ) (b) (cosθ 1 -cosθ 2 )
4 d 4 d
o I o I
(c) (sinθ 1 +sinθ 2 ) (d) (sinθ 1 -sinθ 2 )
4 d 4 d

CENTERS : MUMBAI /DELHI /AKOLA /LUCKNOW /NASHIK /PUNE /NAGPUR /BOKARO /DUBAI # 18
MAGNETISM Rg. 2017 - 2019
3. In the previous question, the magnetic field at P is normal to the plane APB
(a) only if θ1=θ2
(b) only if θ1 and θ2 are very small
(c) only if the length of the sec tion AB is small compared to d.
(d) in all cases

4. ABCD is a square loop made of a uniform conducting wire. A current enters


the loop at A and leaves at D. The magnetic field is
(a) zero only at the centre of the loop
(b) maximum at the centre of the loop
(c) zero at all points outside the loop
(d) zero at all points inside the loop

5. Two long thin wires ABC and DEF are arranged as shown.
They carry currents I as shown. The magnitude of the
magnetic field at O is
 I
(a) zero (b) o
4 a
o I o I
(c) (d)
2 a 2 2 a

6. AB and CD are long straight conductors, distance d apart, carrying a current I.


The magnetic field on BC due to the currents in AB and CD
(a) is zero at all points
(b) is zero only at its midpoint
(c) has different magnitudes at different points
(d) is maximum at its midpoint

7. In the previous question, the magnetic field at the midpoint of BC has magnitude
o I o I o I  I
(a) (b) (c) (d) o
8d 4 d 2 d d
8. A wire carrying a current I is shaped as shown. Section AB is a quarter
circle of radius r. The magnetic field at C is directed
(a) along the bisector of the angle ACB, away from AB
(b) along the bisector of the angle ACB, towards AB
(c) perpendicular to the plane of the paper, directed into the paper
(d) at an angle π/4 to the plane of the paper.
9. In the previous question, the magnitude of the magnetic field at C is
 I  I  I  I
(a) o (b) o (c) o (d) o
2r 4r 8r 4 r

CENTERS : MUMBAI /DELHI /AKOLA /LUCKNOW /NASHIK /PUNE /NAGPUR /BOKARO /DUBAI # 19
MAGNETISM Rg. 2017 - 2019
10.

The wire loop formed by joining two semicircular sections of radii R1 and R2, and centre C, carries a
current I, as shown. The magnetic field at C has magnitude.
 I 1 1   I 1 1   I 1 1   I 1 1 
(a) o    (b) o    (c) o    (d) o   
2  R1 R2  4  R1 R2  2  R1 R2  4  R1 R2 

11. Two parallel beams of protons and electrons, carrying equal currents, are fixed at a separation d. The
protons and electrons move in opposite directions. P is point on a line joining the beams, at a
distance x from any one beam. The magnetic field at P is B. If B is plotted against x, which of the
following best represents the resulting curve?

ANSWERS KEY

1. (C) 2. (A) 3. (D) 4. (A) 5. (C)


6. (C) 7. (D) 8. (C) 9. (C) 10. (D)
11. (C)

10. AMPERE’S LAW


Ampere’s law gives another method to calculate the magnetic
field due to a given current distribution. Consider a current distribution
as shown in the figure. The figure shows a curve enclosing the
currents i1 , i2 and i3 . We assign the direction to these currents by right
hand thumb rule. First we assign a sense to the curve by putting an
arrow on the curve. Then we curl the right hand fingers along the
arrow on the curve, the stretched thumb then give the positive
direction of current. From the figure i1 and i2 are into the plane and
hence in the stretched direction of the thumb and hence are positive
while i3 is out of the plane (hence in the opposite direction of the
thumb stretched) of the paper, hence is negative.

CENTERS : MUMBAI /DELHI /AKOLA /LUCKNOW /NASHIK /PUNE /NAGPUR /BOKARO /DUBAI # 20
MAGNETISM Rg. 2017 - 2019
 
Then take a small length-element d l on the curve and let B be the resultant magnetic field at the position of

d l . Ampere’s law then states that
 

l
B  dl  0ienclosed ………….. (10.1)

Here i is the total current enclosed by the curve which is as per the direction defined earlier is
i = i1  i2  i3 . Any current outside the area is not included in writing the right hand side of the Eqn. 10.1 .
Also the magnetic field on the left hand side of the equation is due to all the currents existing anywhere.
Ampere’s law can be derived from Biot-Savart law and Biot-Savart law can be derived from Ampere’s
law. Hence both are equivalent in scientific content. However, we apply Ampere’s law under certain
symmetrical conditions.

Eqn. 10.1 in a simplified form can be written as ,


Bl   0 i …………. (10.2)
But this equation can be used only under following conditions.
 
(i) At every point of the closed path B is parallel to d l .
(ii) Magnetic field has the same magnitude B at all places on the closed path

Magnetic field at a point due to a thin long, straight current carrying wire

Figure below shows a long, straight current i. We have to calculate the magnetic field at a point P which is at
a distance OP = d from the wire.

Figure shows the situation in the plane perpendicular to the wire and passing through P. The current is
perpendicular to the plane of the diagram and is coming out of it.
Let us draw a circle passing through the point P and with the center at O. We put an arrow to show the
positive sense of the circle. The radius of the circle is OP = d. The magnetic field due to the long, straight
current at any point on the circle is along the tangent as shown in the figure. Same is the direction of the

length element d l there. By symmetry, the magnitude of the magnetic field should be the same at all all
points on the circle. The circulation of the magnetic field along the circle is
 
 B  d l   Bdl
 B  dl  B ( 2d )
The current crossing the area bounded by the circle is i. Thus, from Ampere’s law,

B(2d ) = 0i
0i
or, B = ………….. (10.3)
2d

CENTERS : MUMBAI /DELHI /AKOLA /LUCKNOW /NASHIK /PUNE /NAGPUR /BOKARO /DUBAI # 21
MAGNETISM Rg. 2017 - 2019
Magnetic field created by a thick long current carrying wire.
A long straight wire of radius R carries a steady current I that is uniformly distributed through the cross-
section of the wire.
For finding the behaviour of the magnetic field due to this wire, let us divide
the whole region into two parts. One is r>R and the another is r<R.


For r>R: Let the path of integration be circle as shown in the figure above. From symmetry B must be

constant in magnitude and parallel to d l at every point on this circle. Since total current passing through the
plane of the circle is i , Ampere’s ;law gives,
 
  d l  0i
B
or B l   0i
or B(2πr) = 0i
 i
 B= 0 for (r>R)
2 r

For r<R: Here the current i passing through the plane of circle 2 is less than the total current i . Because the
current is uniform over the cross-section of the wire, the fraction of the current enclosed by the circle 2 must
equal the ratio of the area πr2 enclosed by circle 2 to the cross-sectional area πR2 of the wire.
i r 2

i R 2
r2
i   ( 2 )i
R
Again applying the Ampere’s law to circle 2.
 
  d l  0i
B
r2
B l   0 i B(2πr) =  0 ( )i
R2
 0i
B= ( )r (For r<R)
2R 2
Magnetic field of a solenoid
A solenoid is a long wire wound in the form of helix. With this configuration, a reasonably uniform
magnetic field can be produced in the space surrounded by the turns of wire, which we shall call the interior
of the solenoids when the solenoid carries a current. When the turns are closely spaced, each can be
approximated as a circular loop, and the net magnetic field is the vector sum of the fields resulting from all
the turns. One end of the solenoid behaves like the north pole and the opposite end behaves like the south
pole. As the length of the solenoid increases, the interior field becomes more uniform and the exterior field
becomes weaker. An ideal solenoid is approached when the turns are closely spaced and the length is much
greater than the radius of the turns.
In this case the external field is zero, and the interior field is uniform over the great volume.
We can use Ampere’s law to obtain an expression for the interior magnetic field. In an ideal solenoid.
Figure below shows a longitudinal cross-section of part of such a solenoid carrying a current i. Because the
CENTERS : MUMBAI /DELHI /AKOLA /LUCKNOW /NASHIK /PUNE /NAGPUR /BOKARO /DUBAI # 22
MAGNETISM Rg. 2017 - 2019
 
solenoid is ideal, B in the interior field is uniform and parallel to the axis, and B in the exterior space is
zero. Consider the rectangular path of length l and width w shown in the figure below. We can apply
 
Ampere’s law to this path by evaluating the line integral B  d l over each side of the rectangle.

 
  d l )side3  0
(B as B = 0
   
 (B  d l ) side2,4 0 as B  d l
along these paths
  
 (B  d l ) side1  Bl as B is uniform

and parallel to d l

The integral over the closed rectangular path is therefore,


 
  d l )  Bl
(B
The right side of the Ampere’s law involves the total current passing through the area bounded by the path
of integration.
In this case,
i = (number of turns inside the area)(current through each turn)
= (n l )( i ) where, n is number of turns per unit length
Using Ampere’s law,
 
  d l  0i
B
or B l  (  0 )( nli)
or B =  0 ni …………. (10.4)

MULTIPLE CHOICE QUESTIONS:


1. Current flows through a straight, thin-walled tube of radius r. The magnetic field at a distance x from
the axis of the tube has magnitude B.
(a) B  x, for 0  x  r (b) B  1 / x, for 0  x  r
(c) B  0, for 0  x  r (d) B  0, only for x=0

2. A coaxial cable consists of a thin inner conductor fixed along the axis of a hollow outer conductor.
The two conductors carry equal currents in opposite directions. Let B1 and B2 be the magnetic fields
in the region between the conductors, and outside the conductor, respectively.
(a) B1 ≠ 0, B2 ≠ 0 (b) B1 = B2 = 0 (c) B1 ≠ 0, B2 = 0 (d) B1 = 0, B2 ≠ 0

CENTERS : MUMBAI /DELHI /AKOLA /LUCKNOW /NASHIK /PUNE /NAGPUR /BOKARO /DUBAI # 23
MAGNETISM Rg. 2017 - 2019

3. In the previous question, if both conductors carry equal currents in the same direction,
(a) B1 ≠ 0, B2 ≠ 0 (b) B1 = B2 = 0 (c) B1 ≠ 0, B2 = 0 (d) B1 = 0, B2 ≠ 0

4. In question above let B be the magnetic field at a point between the two conductors, at a distance x
from the axis. Let B2 be the magnetic field at a point outside the outer conductor, at a distance 2x
from the axis
(a) B2 = B1 (b) B1 = 2B2 (c) B1 = 2B1 (d) B2 = 4B1

5. Along straight conductor carrying a current lies along the axis of a ring. The conductor will exert a
force on the ring if the ring.
(a) carries a current (b) has uniformly distributed charge
(c) has nonuniformly distributed charge (d) none of the above

ANSWERS KEY:

1C 2C 3A 4A 5D

CENTERS : MUMBAI /DELHI /AKOLA /LUCKNOW /NASHIK /PUNE /NAGPUR /BOKARO /DUBAI # 24
MAGNETISM Rg. 2017 - 2019

SOLVED EXAMPLES

1. A current path shaped as shown in figure produces a magnetic field at P, the center of the arc. If the
arc subtends an angle of 30° and the radius of the arc is 0.6 m, What are the magnitude and direction
of the field at P if the current is 3.0 A.

Solution: The magnetic field at P due to the straight segments AC and



DE is zero, because d l is parallel to r̂ along these paths, this
 
means that d l  rˆ  0 . Each length element d l along the path
CD as at the same distance R = 0.6 m from P, and the current in

each contributes a field element dB directed into the page at P.

Furthermore, at point on CD, d l is perpendicular to r̂ , hence,

| d l  rˆ | dl . The magnitude of field at P due to the current in an element of length dl,
 idl
dB  0 2
4 R
Because i and R are constants, we can easily integrate this expression over the curved path CD
i i
B  0 2  dl  0 2 (l )
4R 4R
 0i
B  …. (i)
4R 
where we have used the fact that l  R with θ measured in radians. The direction of B

is into the page at P, because d l  rˆ is into the page for every length element.
Substituting the values in Eqn. 1, we have

 3.0   
B  (10 7 )    2.62  10 7 T
 0.6  6 

2. A regular polygon of n sides is formed by bending a wire of total length 2πR which carries
current i.
(a) Find the magnetic field at the center of polygon
(b) Deduce the expression for magnetic field at the center of a circular coil by letting
n  .

Solution: (a) one side of the polygon is,


 2 
 
2R  n   d
a      cot 
n 2 n ( a / 2)
a R   
d  cot  = cot 
2 n n
All sides of the polygon produce the magnetic field at the center in same direction. Hence, net
magnetic field,
B = (n) (magnetic field due to one side)

 i 
= n  0 (sin   sin  )
 4 d 

CENTERS : MUMBAI /DELHI /AKOLA /LUCKNOW /NASHIK /PUNE /NAGPUR /BOKARO /DUBAI # 25
MAGNETISM Rg. 2017 - 2019

2
    
sin   / 
   in       0i    n  n 
B = n  0   tan  2 sin  B=  
 4  R  n  n  2 R  cos( / n)

(b) As n   ,  0 and polygon  circle.
n
sin( / n) /( / n)  1
and cos( / n )  1
 i
B=  0 
 2R 
Which is the expression for the magnetic field at the center of the circular coil.

3. Three infinitely long thin wires, each carrying a current in the same direction, are in the gravity free
space of x-y plane. The central wire is along the y-axis while the other two are along x = ± d.
(a) Find the locus of the points for which the magnetic field B is zero.
(b) If the central wire is displaced in z-direction by a small amount and released, show that it will
execute simple harmonic motion. If the linear density of the wire is λ. Find the frequency of
oscillation.

Solution: (a) Magnetic field will be zero on the y-axis, i.e., x = 0 = z.

Magnetic field cannot be zero in region I and region IV because in region I magnetic field will be in
positive z-direction ( i.e., outside the plane of paper) due to all the three wires, while in region IV
magnetic field will be along negative z-axis( i.e., into the plane of paper) due to all the three wires. It
can be zero only in region II and III.
Let magnetic field is zero on line z = 0 and x = x ( shown as dotted in fig. b). The magnetic field on
this line due to wires 1 and 2 will be along negative z-axis and due to wire 3 along positive z-axis.
Thus,
B1  B2  B3
0 i i  i
or  0  0
2 ( d  x) 2x 2 ( d  x)
1 1 1
 
(d  x) x (d  x)
d
This equation gives x
3
Hence there will be two lines
d d
x and x (z = 0)
3 3
Where magnetic field is zero
CENTERS : MUMBAI /DELHI /AKOLA /LUCKNOW /NASHIK /PUNE /NAGPUR /BOKARO /DUBAI # 26
MAGNETISM Rg. 2017 - 2019

(b) In this part we change our coordinate axis system, just for better understanding.

There are three wires 1, 2 and 3 as shown in the figure above. If we displace the wire 2 towards the
z-axis, then force of attraction per unit length between wires(1 and 2) and (2 and 3) will be given by
 i2
F 0
2 r
The component of F along x-axis will be cancelled out. Net resultant force will be towards negative
z-axis(or mean position) and will be given by
 i2 z
Fnet  2 F cos   2( 0 )
2 r r

0 i2
Fnet  z If z<<d, then
 (z 2  d 2 )
0 i 2
z2  d 2  d 2 and Fnet  ( ) z
 d2

Negative sign implies that Fnet is restoring in nature.


Therefore, Fnet   z
i.e., the wire will oscillate simple harmonically,

Let a be the acceleration of the wire in this position and λ the mass per unit length of the wire then
 i2  i2
Fnet  a  ( 0 2 )  z or a  ( 0 2 )  z
 d  d
therefore frequency of oscillation
1 acceleration
f 
2 displacement
1 a 1 i 0 1 i 0
  or f 
2 z 2 d  2 d 

CENTERS : MUMBAI /DELHI /AKOLA /LUCKNOW /NASHIK /PUNE /NAGPUR /BOKARO /DUBAI # 27
MAGNETISM Rg. 2017 - 2019

4. A thin infinitely large sheet lying in the yz plane carries a current of linear current density λ. The
current is in the negative y direction and λ represents the current per unit length measured along the
z-axis. Find the magnetic field near the sheet.

Solution: To evaluate the line integral in Ampere’s law, let us take a


rectangular path through the sheet as shown in the figure. The
rectangle has dimensions ( l  b ). By symmetry we can argue that
the magnetic field is constant over the sides of length l because
the sheet is infinitely long so it should not vary from point to
point. The two sides of the length b do not contribute to the line

integral because the component of B along the direction of these
paths is zero. The net current passing through the plane is λl.
Applying Ampere’s law,
 

 
B  d l   0i
     
therefore AC  B  d l   B  d l   B  d l    d l   0 (l)
B
CD DE EA

0  Bl  0  Bl  0 l
0 
B  constant
2
The results show that the magnetic field is independent of distance from current sheet. We know that
the electric current due to infinite sheet of charge does not depend on distance from the sheet.

5. Two parallel wires P and Q placed at a separation d = 6 cm carry electric currents


i1 = 5A and i2 = 2A in opposite directions as shown in the figure. Find the point on the line PQ
where the resultant magnetic field is zero.

Solution: At the desired point, the magnetic fields due to two wires must have equal magnitude but opposite
directions. The point should be either to the left of P or to the right of Q. As the wire Q has smaller
current, the point should be closer to Q. Let this point R be at a distance x from Q.
The magnetic field at R due to the current i1 will have magnitude
 0i1
B1 
2 (d  x)
and will be directed downward in the plane of the figure. The field at the same point due to the
current i2 will be
i
B2  0 2
2x
directed upward in the plane of the figure. If the resultant field at R is zero, we should have B1 = B2 ,
so, that
i1 i id ( 2 A)(6cm)
 2 Giving, x  2   4cm
dx x i1  i2 (3 A)

CENTERS : MUMBAI /DELHI /AKOLA /LUCKNOW /NASHIK /PUNE /NAGPUR /BOKARO /DUBAI # 28
MAGNETISM Rg. 2017 - 2019

6. A long wire AB, which is free to move in a vertical


plane and carries a current of 20 A, is in
equilibrium at a height of 0.01 m over another parallel
long wire CD which is fixed in a horizontal plane and
carries a steady current of 30 A, as shown in the figure.
Show that when AB is slightly displaced, it executes a
simple harmonic motion. Find the period of oscillations.
Solution:
Let m be the mass per unit length of the wire AB. At height x above the wire CD, magnetic force per
unit length on wire AB will be given by
 ii
F  0 12 (upwards) …….. (i)
2 x
Weight per unit length of wire AB is

Fg  mg (downwards)
At x = d, wire is in equilibrium, i.e.,
F = Fg
 0 i1i2
or  mg ……… (ii)
2 d
 0 i1i2 mg
or  ……….(iii)
2 d 2 d
when AB is depressed, x decreases therefore, F will increase, while the Fg remains the same. Let
AB is displaced by dx downwards. Differentiating (i) w.r.t. x, we get
 ii
dF =  0 1 22  dx
2 x
i.e., restoring force , dF which is proportional to –dx
Hence the motion of wire is simple harmonic.
From Eqs. (ii) and (iii), we can write
mg
dF = - ·dx
d
g
Acceleration of wire a = - dx
d
Hence period of oscillations
dx displacement d 0.01
T = 2  2 T = 2  2 or T = 0.2 s
a acceleration g 9.8

7.
A charged particle (q, m) enters a uniform

magnetic field B at an angle α as shown in the
figure with speed v0 . Find,
(a) the angle β at which it leaves the magnetic
field
(b) time spent by the particle in the magnetic
field and
(c) the distance AC.
Solution: (a) Here, velocity of the particle is in plane of paper while magnetic field is perpendicular to
the paper inwards. i.e., angle between v and B is 90 deg. So the path is a circle. The radius of
the circle is,

CENTERS : MUMBAI /DELHI /AKOLA /LUCKNOW /NASHIK /PUNE /NAGPUR /BOKARO /DUBAI # 29
MAGNETISM Rg. 2017 - 2019

mv0
r
qB
O is the center of the circle. In triangle AOC,
 OCD =  OAD
90° – β = 90° - α
therefore β= α
(b)  COD =  DOA = α ( as  OCD =  OAD = 90° - α )
 AOC = 2α
2mv0
or Length ABC = r(2 α) = 
qB

ABC 2m
therefore t ABC  
v0 Bq

2mv0
(c) Distance AC = 2 (AD) = 2(r sin α) =  sin 
qB
 
8. A particle of specific charge α enters a uniform magnetic field B = -B0 k̂ with velocity v  v 0 ˆi from
the origin. Find the time dependency
 of velocity and position of the particle.

Solution: First of all the angle between B and v will decide the path of the particle. Here the angle is
90 deg. hence the path is the circle. The sense of rotation of the particle will be anti-clockwise
because at the origin the magnetic force is along positive y-direction(which can be seen from
fleming’s right hand rule).
v0
θ = ωt = B0t and r 
B 0

With reference to the figure,


Velocity of the particle at any time t is,

v(t)  v x ˆi  v yˆj  v 0 cos (i)
ˆ  v sin (ˆj)
0

v(t)  v x i  v y j  v 0 cos(B0t)(i)  v 0 sin(B0t)(ˆj)
ˆ ˆ ˆ
or
Position of particle at time t is,

r (t)  xiˆ  yjˆ  r sin (i)
ˆ  (r  r cos )(ˆj)
Substituting the values of r and θ, we have
 v ˆ  (1  cos(B t))(ˆj)]
r (t)  0 [sin(B0 t)(i) 0
B0 
CENTERS : MUMBAI /DELHI /AKOLA /LUCKNOW /NASHIK /PUNE /NAGPUR /BOKARO /DUBAI # 30
MAGNETISM Rg. 2017 - 2019


9. A particle of charge q and mass m is projected from the origin with velocity v  v 0 ˆi in a non-

uniform magnetic field B = -B0x k̂ . Here v0 and B0 are positive constants of proper dimensions.
Find the maximum positive x coordinate of the particle during its motion.
Solution: Magnetic field is along negative z-direction. So in the co-ordinate axis shown in figure, it is
Perpendicular to paper inwards. Magnetic force on the particle at origin is along positive y-direction.
So it will rotate in x-y plane as shown. The path is not a perfect circle as the magnetic field is non-
uniform. Speed of the particle in magnetic field remains constant. Magnetic force is always
perpendicular to velocity. Let at point P(x,y) its velocity vector makes an angle θ with positive x-

axis. Then magnetic force F will be at an angle θ with positive y direction. So

ay = (F/m) cos θ
where magnetic force F is F = Bqv0 sin 90
dv y ( B0 x)(qv0 cos  )

dt m
dv
 y   dx  (B0 x)(qv 0 cos )
   
 dx   dt  m
dx
Here = vx = v0 cos 
dt
v0 xmax
dv y  B0 q  B q B q  x 2 max 
= x  dv y   0   xdx v0 =  0   
dx  m  0  m  0  m   2 
2mv0
xmax =
B0 q

10. A flat dielectric disc of radius R carries an excess charge on its surface. The surface charge density is
σ. The disc rotates about an axis perpendicular to its plane passing through the center with angular
velocity ω. Find the torque on the disc if it is placed in a uniform magnetic field B directed
perpendicular to the rotation axis.
Solution: consider an annular ring of radius r and of thickness dr on this
disc.

Charge on this ring,


dq = (σ) (2πr dr)
As ring rotates with angular velocity ω, the equivalent current is,
i = (dq) (frequency)

= (σ) (2πr dr) i = σ ω r dr
2
or
Magnetic moment of this annular ring,
CENTERS : MUMBAI /DELHI /AKOLA /LUCKNOW /NASHIK /PUNE /NAGPUR /BOKARO /DUBAI # 31
MAGNETISM Rg. 2017 - 2019

M = iA = (σ ω r dr)( πr2) (along the axis of rotation)

Torque on this ring, d = MB sin 90° = (σ ω r3 B) πdr


R R
 BR 4
Therefore total torque on the disc is,    d  (B )  r 3dr =
0 0
4

11. A uniform constant magnetic field B is directed at an angle of 45° to the x-axis in x-y plane. PQRS is
a rigid square wire frame carrying a steady current Io, with its center at the origin O. At time t = 0,
the frame is at rest in the position shown in the figure with its sides parallel to x and y axes. Each
side of the frame is of mass M and length L.

(a) What is the torque  about O acting on the frame due to the magnetic field?
(b) Find the angle by which the frame rotates under the action of this torque in a short interval of
time Δt, and the axis about which this rotation occurs(Δt is so short that any variation in the
torque in this interval may be neglected). Given: the moment of inertia of the frame about an
axis through its centre perpendicular to its plane is (4/3)ML2

Solution: Magnetic moment of the loop,



  (iA)kˆ  (I0 L2 )kˆ
 B ˆ ˆ
Magnetic field, B = ( B cos 450 )iˆ  ( B sin 450 ) ˆj  (i  j )
2
(a) Torque acting on the loop,
  
ˆ   B (iˆ  ˆj) 
    B  (I0 L2 k)  
 2 
2
I LB ˆ ˆ
 0 ( j  i) or   I 0 L2 B
2
(b) Axis of rotation coincides with the torque and since torque is in ( ˆj  iˆ) direction or parallel to QS.
Therefore, the loop will rotate about an axis passing through Q and S as shown alongside:

Angular acceleration, 
I
where I is moment of inertia of loop about QS.
I QS  I PR  I ZZ (from theorem of perpendicular axis)
4  3 I0 B
But I QS  I PR 2 I QS  I ZZ  ML2  =
3 I 2 M
1 3 I0B
Angle by which the frame rotates in time Δ t is Δθ =  ( t 2 ) or Δθ = ( t ) 2
2 4 M

CENTERS : MUMBAI /DELHI /AKOLA /LUCKNOW /NASHIK /PUNE /NAGPUR /BOKARO /DUBAI # 32
MAGNETISM Rg. 2017 - 2019

12. Consider a coaxial cable which consists of an inner wire of radius a surrounded by an outer shell of
inner and outer radii b and c respectively. The inner wire carries a current i0 and the outer shell
carries a equal current in opposite direction. Find the magnetic field at distance x from the axis where
(a) x < a, (b) a < x < b (c) b< x < c (d) x > c. Assume that the current density is uniform in
the inner wire and also uniform in the outer shell.

Solution:

A cross section of the cable is shown in the figure. Draw a circle of radius x with the center at the
axis of the cable. The parts a, b, c, d of the figure correspond to the four parts of the problem. By
symmetry, the magnetic field at each point of the circle will have the same magnitude and will be
tangential to it. The circulation of B along each circle is, therefore,
 
  d l B2x
B
in each of the four parts of the figure.

(a) The current enclosed within the circle in part (a) is


i0 i
2
 x 2  02 x 2
a a
   i x2 ix
Ampere’s law  B  d l  0i gives B 2x = 0a02 or, B  0 02
2a
The direction will be along tangential to the circle.
(b) The current enclosed within the circle in part (b) is i0 so that
i
B 2x =  0i0 or, B 00
2x
(c) The area of cross section of outer shell is c 2  b 2 . The area of cross-section of outer shell within
i (x2  b2 )
the circle in part (c) of the figure is x 2  b 2 . Thus the current through this part is 0 2 . This
c  b2
is in the opposite direction to the current i0 in the inner wire. Thus the net current enclosed by the
circle is
i ( x2  b2 ) i0 (c 2  x 2 )
i0  0 2 =
c  b2 c2  b2
 i (c 2  x 2 )  0i0 (c 2  x 2 )
From Ampere’s law, B 2x = 0 0 2 or, B 
c  b2 2x(c 2  b 2 )
(d) The net current enclosed by the circle in part d of the figure is zero and hence

B 2x = 0 or, B=0

CENTERS : MUMBAI /DELHI /AKOLA /LUCKNOW /NASHIK /PUNE /NAGPUR /BOKARO /DUBAI # 33
MAGNETISM Rg. 2017 - 2019

EXERCISE - 1

Q.1 A current of i ampere is flowing through each of the bent wires as shown in the fig. the magnitude of the
magnetic field at O is
 0i  1 2   0i  1 3 
(A)    (B)   
4  R R  4  R R 
 0i  1 3   0i  1 3 
(C)    (D)   
8  R 2R   8  R R 

Q.2 Net magnetic field at the centre of the circle O due to a current carrying
loop as shown in figure is ( < 180°)
(A) zero
(B) perpendicular to paper inwards
(C) perpendicular to paper outwards
(D) is perpendicular to paper inwards if   90° and perpendicular to paper outwards if 90°<180°

Q.3 The magnetic field due to a current carrying square loop of side a at a point located
symmetrically at a distance of a/2 from its centre (as shown is)
2  0i 0 i 2  0i
(A) (B) (C) (D) zero
3 a 6 a 3 a

Q.4 A charge particle A of charge q = 2 C has velocity v = 100 m/s. When it passes through
point A and has velocity in the direction shown. The strength of magnetic field at point B
due to this moving charge is (r = 2 m).
(A) 2.5 T (B) 5.0 T (C) 2.0 T (D) None

Q.5 Three rings, each having equal radius R, are placed mutually perpendicular to each
other and each having its centre at the origin of co-ordinate system. If current I is
flowing through each ring then the magnitude of the magnetic field at the common
centre is
0I 0I 0I
(A) 3
2R
(B) zero 
(C) 2  1 
2R

(D) 3  2 
2R

Q.6 Two concentric coils X and Y of radii 16 cm and 10 cm lie in the same vertical plane containing N-S direction.
X has 20 turns and carries 16 A. Y has 25 turns & carries 18A. X has current in anticlockwise direction and
Y has current in clockwise direction for an observer, looking at the coils facing the west. The magnitude of net
magnetic field at their common centre is
(A) 5 × 10–4 T towards west (B) 13 × 10–4 T towards east
–4
(C) 13 × 10 T towards west (D) 5 × 10–4 T towards east

Q.7 A uniform beam of positively charged particles is moving with a constant velocity parallel to another beam of
negatively charged particles moving with the same velocity in opposite direction separated by a distance d. The
variation of magnetic field B along a perpendicular line draw between the two beams is best represented by

(A) (B) (C) (D)

CENTERS : MUMBAI / DELHI / AKOLA / LUCKNOW / NASHIK / PUNE / NAGPUR / BOKARO /DUBAI # 34
MAGNETISM Rg. 2017 - 2019


Q.8 The dimension of where  is permeability &  is permittivity is same as :

(A) Resistance (B) Inductance (C) Capacitance (D) None of these

Q.9 A current I flows around a closed path on the horizontal plane of the circle as shown in
the figure. The path consists of eight arcs with alternating radii r and 2r. Each segment
of arc subtends equal angle at the common centre P. The magnetic field produced by
current path at point P is
3 0I
(A) ; perpendicular to the plane of the paper and directed inward.
8 r
3 0I
(B) ; perpendicular to the plane of the paper and directed outward.
8 r
1 0I
(C) ; perpendicular to the plane of the paper and directed inward.
8 r
1 0I
(D) ; perpendicular to the plane of the paper and directed outward..
8 r

Q.10 Infinite number of straight wires each carrying current I are equally
spaced as shown in the figure. Adjacent wires have current in opposite
direction. Net magnetic field at point P is
 0 I ln 2  0 I ln 4
(A) k̂ (B) k̂
4 3 a 4 3 a
 0 I ln 4
(C) ( k̂ ) (D) Zero
4 3 a

Q.11 A direct current is passing through a wire. It is bent to form a coil of one turn. Now it is further bent to form a
coil of two turns but at smaller radius. The ratio of the magnetic induction at the centre of this coil and at the
centre of the coil of one turn is
(A) 1 : 4 (B) 4 : 1 (C) 2 : 1 (D) 1 : 1

Q.12 Two mutually perpendicular conductors carrying currents I1 and I2 lie in one plane. Locus of the point at which
the magnetic induction is zero, is a
(A) circle with centre as the point of intersection of the conductor.
(B) parabola with vertex as the point of intersection of the conductors
(C) straight line passing through the point of intersection of the conductors.
(D) rectangular hyperbola

Q.13 Find the magnetic field at P due to the arrangement shown


 0i  1  2 0i  0i  0i  1 
(A) 1    (B)  (C)  (D) 1  
2 d  2 2 d 2 d 2 d  2

Q.14 Equal current i is flowing in three infinitely long wires along positive x, y and z directions. The magnitude field
at a point (0, 0, –a) would be:
0i  0i  i  i
(A) ( ĵ  î ) (B) ( î  ĵ) (C) 0 ( î  ĵ) (D) 0 (î  ˆj  k̂ )
2a 2a 2a 2a

CENTERS : MUMBAI / DELHI / AKOLA / LUCKNOW / NASHIK / PUNE / NAGPUR / BOKARO /DUBAI # 35
MAGNETISM Rg. 2017 - 2019

Q.15 A thin, straight conductor lies along the axis of a hollow conductor of radius R. The two carry equal currents in
the same direction. The magnetic field B is plotted against the distance r from the axis. Which of the following
best represents the resulting curve?

(A) (B) (C) (D)

Q.16 A long thin walled pipe of radius R carries a current I along its length. The current density is
uniform over the circumference of the pipe. The magnetic field at the center of the pipe due
to quarter portion of the pipe shown, is
0I 2 0I 2 0I 2
(A) 2 (B) 2 (C) (D) None
4 R  R 2R

Q.17 Two very long straight parallel wires, parallel to y-axis, carry currents 4I and I, along +y direction and –y direction,
respectively. The wires are passed through the x-axis at the points (d, 0, 0) and (– d, 0, 0) respectively. The graph of
magnetic field z-component as one moves along the x-axis from x = – d to x = +d, is best given by

(A) (B) (C) (D)

Q.18 A long straight wire, carrying current I, is bent at its midpoint to form an angle of 45°. •P I
R 45°
Induction of magnetic field at point P, distant R from point of bending is
equal to :

(A)
 
2 1  0I
(B)
 
2  1 0I
(C)
 
2 1  0I
(D)
 
2  1  0I
4 R 4R 4 2R 4 2 R

Q.19 A hollow cylinder having infinite length and carrying uniform current per unit length  along the
circumference as shown. Magnetic field inside the cylinder is
 0
(A) (B) 0 (C) 20 (D) none
2

Q.20 A long straight metal rod has a very long hole of radius ‘a’ drilled parallel to the rod axis as shown in the figure.
If the rod carries a current ‘i’ find the value of magnetic induction on the
axis of the hole, where OC = c
.a

(A)
 0ic
( b 2  a 2 )
(B)
 0ic
2 ( b 2  a 2 )
C
.
O
b

 0i( b 2  a 2 )  0 ic
(C) (D)
2 c 2 a 2 b 2
Q.21 Two long conductors are arranged as shown in fig. to form overlapping
cylinders, each of radius r, whose centers are separated by a distance d. Current
of density J flows into the plane of the page along the shaded part of one
conductor and an equal current flows out of the plane of the page along the
shaded portion of the other, as shown. What are the magnitude and direction
of the magnetic field at point A?
(A) (0/2)dJ, in the +y-direction (B) (0/2)d2/r, in the +y-direction
(C) (0/2)4d2J/r, in the –y-direction (D) (0/2)Jr2/d, in the –y-direction
CENTERS : MUMBAI / DELHI / AKOLA / LUCKNOW / NASHIK / PUNE / NAGPUR / BOKARO /DUBAI # 36
MAGNETISM Rg. 2017 - 2019

Q.22 An electron is moving along positive x-axis. A uniform electric field exists towards negative y-axis. What
should be the direction of magnetic field of suitable magnitude so that net force on the electron is zero.
(A) positive z- axis (B) negative z-axis (C) positive y-axis (D) negative y-axis

Q.23 A particle of charge q and mass m starts moving from the origin under the action of an electric field E  E 0ˆi
 
and B  B0 iˆ with velocity v  v 0ˆj . The speed of the particle will become 2v0 after a time,
2mv0 2 Bq 3 Bq 3 m v0
(A) t = (B) t = mv (C) t = (D) t =
qE 0 m v0 qE
Q.24 An electron is projected with velocity v0 in a uniform electric field E perpendicular to the field. Again it is
projetced with velocity v0 perpendicular to a uniform magnetic field B. If r1 is initial radius of curvature just after
entering in the electric field and r2 is initial radius of curvature just after entering in magnetic field then the ratio
r1 r2 is equal to,
Bv 02 B Ev0 Bv 0
(A) (B) (C) (D)
E E B E

Q.25 ˆ
A uniform magnetic field B  B0 j exists in a space. A particle of mass m and charge q is projected towards
negative x-axis with speed v from the a point (d, 0, 0). The maximum value v for which the particle does not
hit y-z plane is
2 Bq Bqd Bq Bqd
(A) (B) (C) (D)
dm m 2dm 2m

Q.26 Two protons move parallel to each other, keeping distance r between them, both moving with same velocity

V . Then the ratio of the electric and magnetic force of interaction between them is
(A) c 2 V 2 (B) 2c 2 V 2 (C) c 2 2V 2 (D) None


Q.27 A charged particle of specific charge  is released from origin at time t = 0 with velocity V  Voˆi  Voˆj in
 
magnetic field B  Bo ˆi . The coordinates of the particle at time t  are (specific charge = q/m)
Bo 
 Vo 2Vo  Vo    Vo 
(A)  2B  , B , B   (B)  2B  , 0, 0 
 o o o   o 
 2Vo Vo    Vo  2Vo 
(C)  0, B  , 2B   (D)  B  , 0,  B  , 
 o o   o o 

Q.28 Three ions H+, He+ and O+2 having same kinetic energy pass through a region in which there is a uniform
magnetic field perpendicular to their velocity, then :
(A) H+ will be least deflected. (B) He+ and O+2 will be deflected equally.
+2
(C) O will be deflected most. (D) all will be deflected equally.

Q.29 An electron having kinetic energy T is moving in a circular orbit of radius R perpendicular to a uniform magnetic

induction B . If kinetic energy is doubled and magnetic induction tripled, the radius will become,
3R 3 2 4
(A) (B) R (C) R (D) R
2 2 9 3

CENTERS : MUMBAI / DELHI / AKOLA / LUCKNOW / NASHIK / PUNE / NAGPUR / BOKARO /DUBAI # 37
MAGNETISM Rg. 2017 - 2019

Q.30 A n electron (mass = 9.1 × 1031 ; charge =  1.6 × 1019 C) experiences no deflection if subjected to an
electric field of 3.2 × 105 V/m and a magnetic field of 2.0 × 103 Wb/m2 . Both the fields are normal to the path
of electron and to each other . If the electric field is removed, then the electron will revolve in an orbit of radius

(A) 45 m (B) 4.5 m (C) 0.45 m (D) 0.045 m


 
Q.31 A charged particle moves in a magnetic field B  10 ˆi with initial velocity u  5iˆ  4ˆj . The path of the particle
will be
(A) straight line (B) circle (C) helical (D) none

 
Q.32 An electron experiences a force 4.0 î  3.0 ˆj × 10–13 N in an uniform magnetic field when its velocity is
 
2.5 k̂ 10 ms–1. When the velocity is redirected and becomes 1.5 î  2.0 ĵ  107 ms–1, the magnetic force
7


on the electron is zero. The magnetic field vector B is :
(A) – 0.075 î  0.1 ĵ (B) 0.1 î  0.075 ĵ (C) 0.075 î  0.1 ˆj  k̂ (D) 0.075 î  0.1ˆj

Q.33 A mass spectrometer is a device which select particle of equal mass. An iron with electric charge q > 0 and
mass m starts at rest from a source S and is accelerated through a potential difference V. It passes through a

hole into a region of constant magnetic field B perpendicular to the plane of the paper as shown in the figure.
The particle is deflected by the magnetic field and emerges through the bottom hole at a distance d from the top
hole. The mass of the particle is
qBd qB2d 2 qB2d 2 qBd
(A) (B) (C) (D)
mV 4V 8V 2mV

CENTERS : MUMBAI / DELHI / AKOLA / LUCKNOW / NASHIK / PUNE / NAGPUR / BOKARO /DUBAI # 38
MAGNETISM Rg. 2017 - 2019

EXERCISE – 2
(One or More than one option may be correct)
1. A conductor of mass m and length  , carrying current I is placed on smooth inclined making angle
 with horizontal. A magnetic field B is directed vertically upwards. Then for equilibrium of
conductor tan  is given by
2mg mg mg Bi
(a) (b) (c) (d)
Bi Bi 2 Bi mg

2. The magnetic field at centre of a hexagonal coil of side  carrying a current i is r
3 0 I 0 I 0 I
(a) (b) (c) (d) zero O
 4  3

3. A conductor AB of length L carrying a current I1 is placed perpendicular Y


to a long straight conductor x-y carrying a current I2, as shown in the
figure. The force on AB has magnitude is I1
II II I2 A B
(a) 0 1 2 ln 2 (b) 0 1 2 ln 3
2 2 L/2 L
30 I1I 2 2 0 I1I 2
(c) (d) X
2 3

4. A current i flows along a thin wire shaped as shown in figure. The radius of the
curved part of the wire is r. The field at the centre O of the coil is: O
 i  i i
(a) 0  (b) 0 (c) 0 (d) 0 (3  4) 90o
4 r 2 r 2 r 8 r
A B

5. A charged particle is released from the rest in a region of steady and uniform electric and magnetic
fields which are parallel to each other. The particle will move in a
(a) Straight line (b) Circle (c) Helix (d) Cycloid

6. A proton moving with a constant velocity passes through a region of space without any change in its
velocity. If E and B represent the electric and magnetic fields respectively, this region of space may
have
(a) E = 0, B = 0 (b) E = 0, B  0 (c) E  0, B = 0 (d) E  0, B  0
7. A coaxial cable consists of a thin inner current carrying conductor fixed along the axis of a hollow
current carrying conductor. Let B1 and B2 be the magnetic fields in the region between the
conductors, and outside the conductor, respectively
(a) B1  0, B2  0, for conductors carrying equal currents in opposite directions
(b) B1  0, B2 = 0, for conductors carrying equal currents in opposite directions
(c) B1  0, B2  0, for conductors carrying equal currents in same directions
(d) B1 = B2 for conductors carrying equal currents in same directions at distance x and 2x from the
axis respectively. 2x > the radius of the outer cable.

CENTERS : MUMBAI /DELHI /AKOLA /LUCKNOW /NASHIK /PUNE /NAGPUR /BOKARO /DUBAI # 39
MAGNETISM Rg. 2017 - 2019

P Q
8. A charged particle having kinetic energy k and charge q enters into the region of a
uniform magnetic field between two plates P and Q as shown in Figure. The
charged particle just misses hitting the plate Q. The magnetic field in the region
between the two plates is: q
(a) mk/qa (b) 2mk/qa (c) √(mk)/qa (d) √(2mk)/qa
a
 
9. Electric and magnetic field are directed as E0 i and B0 k , a particle of mass m and charge q is
 
released from position (0, 2, 0) from rest. The velocity of that particle at (x, 5, 0) is (5 i  12 j ) the
value of x will be
169 m 25m 25m 144m
(a) (b) (c) (d)
2qE0 2qE0 12qE0 12 qE0

10. A conductor AB of length a, carrying current I  , is placed M 


perpendicular to a long straight conductor LM carrying a current k
B I' A 
I as show in figure. The force on AB will be I j
  a 
(a) in the k (b) in the (k ) a i
L
 II   II  2
(c) of magnitude 0 ln 2 (d) of magnitude 0 ln 3
2 2
11. A long straight wire of radius R carries a current distributed uniformly over its cross-section. The
magnitude of the magnetic field is
(a) minimum at the axis of the wire (b) maximum at the surface of the wire
(c) maximum at the axis of the wire (d) minimum at the surface of the wire
12. A long straight wire carries a current along the Z-axis. One can find two points in the X-Y plane such
that
(a) the magnetic fields are equal
(b) the directions of the magnetic fields are the same
(c) the magnitudes of the magnetic fields are equal
(d) the field at one point is opposite to that the other point

13. A hollow tube is carrying an electric current along its length distributed uniformly over its surface.
The magnetic field
(a) increases linearly from the axis to the surface (b) is constant inside the tube
(c) is zero at the axis (d) is zero just outside the tube

14. The magnetic field at the origin due to a current element id  placed at position r is
       
 0i d   r 0 i r  d   0i r  d  0 i d   r
(a) (b)  (c) (d) 
4 r 3 4 r 2 4 r 3 4 r 3

15. Two charged particles M and N are projected with the same velocity in a uniform magnetic field as
shown. Then, M and N, respectively, can be
M
x x x
N

x x x
(a) an  -particle and a proton (b) a deutron and a proton
(c) a deutron and an  -particle (d) a proton and an  -particle

CENTERS : MUMBAI /DELHI /AKOLA /LUCKNOW /NASHIK /PUNE /NAGPUR /BOKARO /DUBAI # 40
MAGNETISM Rg. 2017 - 2019

16. An observer A and a charge Q are fixed in a stationary


frame F1. An observer B is fixed in a frame F2, which is A B
moving with respect to F1. F1 Q F2
(a) Both A and B will observe electric fields
(b) Both A and B will observe magnetic fields
(c) Neither A nor B will observe magnetic field
(d) B will observe a magnetic field, but A will not.

17. A long straight wire carries a current along the x-axis. Consider the points A (0, 1, 0), B (0, 1, 1) C
(1, 0, 1) and D (1, 1, 1). Which of the following pairs of points will have magnetic fields of the same
magnitude?
(a) A and B (b) A and C (c) B and C (d) B and D

18. In the previous question, if the current is i and the magnetic field at D has magnitude B,
i i
(a) B  0 (b) B  0
2 2 2 3
(c) B is parallel to the z-axis (d) B makes an angle of 45 o with the x-y plane

19. L is a circular ring made of a uniform wire. Current enters and leaves L
the ring through straight conductors which, if produced, would have
C
passed through the centre C of the ring. The magnetic field at C
(a) due to the straight conductors is zero 
(b) due to the loop is zero i
(c) due to the loop is proportional to 
(d) due to the loop is proportional to (   ) i

20. In the loops shown, all curved sections are either semicircles or quarter circle. All the loops carry the
same current. The magnetic fields at the centres have magnitudes B1, B2, B3 and B4

a
B1 a a
a
b b
b B2 B3 b B4

(a) B4 is maximum (b) B3 is minimum


(c) B4 > B1 > B2 > B3 (d) B1 > B4 > B3 > B2

CENTERS : MUMBAI /DELHI /AKOLA /LUCKNOW /NASHIK /PUNE /NAGPUR /BOKARO /DUBAI # 41
MAGNETISM Rg. 2017 - 2019

EXERCISE – 3
COMPREHENSION TYPE
Comprehension – I

Analog voltmeters and ammeters work by measuring the


torque exerted by a magnetic field on a current carrying Scale
coil. The reading is displayed by means of the deflection
Coil
of a pointer over a scale. The adjacent figure shows the Pointer

Permanent Magnet
essentials of a galvanometer, on which both analog
ammeters and analog voltmeters are based. Assume that
the coil is 2.1 cm high, 1.2 cm wide, has 250 turns and is
mounted so that it can rotate about an axis (into the
N S
page) in a uniform radial magnetic field with B = 0.23 Sp
T. For any orientation of the coil, the net magnetic field
through the coil is perpendicular to the normal vector of
the coil (and thus parallel to the plane of coil). A spring
SP provides a counter torque that balances the magnetic Soft iron coil
torque, so that a given steady current I in the coil results
in a steady angular deflection  . The greater the current , greater the deflection , and thus greater the
torque required for the spring . A current of 100  A produces an angular deflection of 28o.

1. What must be the torsional constant K of the spring?


(a) 2.6 108 Nm/degree (b) 5.2 10 8 Nm/degree
(c) 2.6 104 Nm/degree (d) 5.2 10 4 Nm/degree

2. If we reduce the value of this K to half of its value, then the deflection would be
(a) 28 o (b) 56 o (c) 14 o (d) none

3. If the value of magnetic field is put equal to 0.69 T and K = 15.6  10-8 Nm/degree. Then, the
deflection would be
(a) < 28o (b) = 28o (c) 14 o (d) none

Comprehension – II
To know the structure of matter on smallest scale, we allow an
energetic charged particle to slam into a solid target or allow two energetic
charged particles to collide head on. We use cyclotrons to give very high
kinetic energy to the charged particles. We arrange for the proton to s

circulate in a magnetic field and give it a modest electrical kick once per
revolution. Two hollow D shaped objects, called ‘dees’, are part of an
electrical oscillator that alternates the electric potential difference across the
gap between the ‘dees’, back and forth. The ‘dees’ are immersed in a
magnetic field whose direction is out of the plane of page and that is set up by a large electromagnet.
Suppose a proton, injected by a source S at the center of the cyclotron, initially moves toward a
negatively charged dee. It will accelerate toward this dee and enter it. Once inside, it is shielded from
electric fields by the copper walls of the dee; that is, the electric field does not enter the dee. The
magnetic field, however, is not screened by the copper dee. So, the proton moves in a circular path
mv
whose radius, which depends on speed, is given by r 
qB

CENTERS : MUMBAI /DELHI /AKOLA /LUCKNOW /NASHIK /PUNE /NAGPUR /BOKARO /DUBAI # 42
MAGNETISM Rg. 2017 - 2019

The key to the operation of cyclotron is that the frequency f at which the proton circulates in the field
and that does not depend on its speed must be equal to the fixed frequency fosc of the electrical
oscillator. The resonance condition is given by qB = 2 m fosc
4. If a proton circulates 200 times in a magnetic field of 1.5 T and receives an energy boost of 100 KeV
every time it completes an orbit, it will end up with a kinetic energy of
(a) 100 KeV (b) 20 MeV (c) 40 MeV (d) 30 MeV

5. Suppose a cyclotron is operated at an oscillator frequency of 24 MHz and has a dee of radius 26.5
cm. Then, what is the magnitude of the magnetic field needed for deuterons to be accelerated in the
cyclotrons?
(a) 1.57 T (b) 3.14 T (c) 4.71 T (d) none of these

6. What is the resulting kinetic energy of deuterons?


(a) 3.99 1012 J (b) 2.7 1012 J (c) 10 12 eV (d) none of these

Comprehension – III
The basic principle underlying the Hall effect is the Lorentz
force. When an electron moves along a direction Y
B
perpendicular to an applied magnetic field, it experiences a
force acting normal to both the directions and moves in z1
response to this force and the force exerted by the internal I
electric field. For an n-type, bar-shaped semiconductor, the y
1
carriers are predominantly electrons of bulk density n. We
assume that the constant current I flows along the x-axis
from left to right in the presence of a magnetic field toward
y-axis. Electrons subjected to the lorentz force initially drift away from the current line toward the
negative z-axis, resulting in an excess surface electrical charge on the sides of the sample. This
charge results in the hall voltage, a potential drop across the two sides of the sample. The
transverse voltage is the hall voltage vH and its magnitude is equal to IB/qnd, where I is the current,
B is the magnetic field, d is the sample thickness and q is the elementary charge. Given z1 = 11.8 mm
and y1 = 0.23 mm and a current of 120 A is flowing in the x-direction in a uniform magnetic field B
= 0.95 T. If electron density is 5.85 10 28 /m3, then

7. Magnitude of the drift velocity of electron is


(a) 2.35 mm/s (b) 4.7 mm/s (c) 7.05 mm/s (d) 9.4 mm/s

8. What is the magnitude and direction of electric field?


(a) 4.5 103 V/m in + z dir (b) 45 10 3 V/m in – z dir
(c) 2.25 10 3 V/m in + x dir (d) 2.25 10 3 V/m in – x dir

9. What is the value of hall emf ?


(a) 53 mV (b) 53  V (c) 5.3 mV (d) 5.3  V

CENTERS : MUMBAI /DELHI /AKOLA /LUCKNOW /NASHIK /PUNE /NAGPUR /BOKARO /DUBAI # 43
MAGNETISM Rg. 2017 - 2019

MATRIX MATCH TYPE

10. A charged particle is moving in a circular path in uniform magnetic field with speed v. Match the
following.
Column – I Column – II

(A) Equivalent current due to motion of charged (p) is proportional to v


particle
(B) Magnetic Moment (q) is proportional to v2

(C) Magnetic field at centre of circle due to motion (r) is proportion to v0


of charged particle.
(D) Radius of circular path (s) none of these

11. In the moving coil galvanometer


N = number of turns; A = cross sectional area; B = Magnetic field;
C = Torsional constant; i = current; R = resistance
Column – I Column – II

(A) Torque acting on the coil (p) NAB/C


(B) Deflection produced (q) NAB/CR
(C) Current sensitivity (r) NIAB
(D) Voltage sensitivity (s) NABI/C

Assertion Reasoning Type:


(A) Statement-1 is True, Statement-2 is True; Statement-2 is a correct explanation for
Statement-1.
(B) Statement-1 is True, Statement-2 is True; Statement-2 is NOT a correct explanation for
Statement-1.
(C) Statement-1 is True, Statement-2 is False.
(D) Statement-1 is False, Statement-2 is True.

12. Statement-1: The magnetic field at the centre of a circular loop carrying current is zero.
Statement-2: Due to symmetry, the field produced by the diametrically opposite points add to each
other.

13. Statement-1: Ampere’s law cannot be used to determine magnetic field due a current carrying wire
of finite length.
Statement-2: Ampere’s law is not valid for wires of finite length.

CENTERS : MUMBAI /DELHI /AKOLA /LUCKNOW /NASHIK /PUNE /NAGPUR /BOKARO /DUBAI # 44
MAGNETISM Rg. 2017 - 2019

EXERCISE – 4
(SUBJECTIVE PROBLEMS)

1. Find the magnetic induction of the field at the point O of loop with the current I, whose shaped is
illustrated.
(a) in fig, radii a and b, as well as the angle  are known;
(b) in fig, the radius a and the side b are known.

I b
b I
 b
a a
O
O

2. A current I = 5.0 A flows along a thin wire shaped as shown in fig. The radius of a curved part of the
wire is equal to R = 120 mm, the angle 2  90o , Find the magnetic induction of the point O.

O I

2 R

3. Find the magnetic induction at the centre of a rectangular wire frame whose diagonal is equal to
d = 16 cm and the angle between the diagonal is equal to   30; the current flowing in the frame
equal I = 5.0 A.

4. Find the magnetic induction at the point O if the wire carrying a current I = 8.0 A has the shaped
shown in fig, The radius curved part of the wire is R = 100 mm, the linear parts of the wire are very
long.
z
z z

R R R
O y O
y O y

x x x

5. Two mutually perpendicular conductors carrying currents I1 and I2 lie in one plane. Find the locus of
points at which the magnetic induction is zero.
I2

II I
I1
III IV

CENTERS : MUMBAI /DELHI /AKOLA /LUCKNOW /NASHIK /PUNE /NAGPUR /BOKARO /DUBAI # 45
MAGNETISM Rg. 2017 - 2019
y
D (  a , a) A( a, a)
6. 4 long wires each carrying current I as shown in the figure are placed
 
at the points A, B, C and D. Find the magnitude and direction of:
(a) Magnetic field at the centre of the square. x
(b) Magnitude of force per metre acting on wire at point D.
 
C ( a, a) B( a, a)
7. A very long wire carrying a current I = 5.0 A is bent at right angles. Find the magnetic induction at a
point lying on a perpendicular to the wire, drawn through the point of bending, at a distance 1 = 35
cm from it.

8. A non conducting thin disc of radius R charged uniformly over one side with surface charge density
 rotates about its axis with an angular velocity  . Find:
(a) the magnetic induction at the centre of the disc;
(b) the magnetic moment of the disc.

9. A non-conducting sphere of radius R = 50 mm charged uniformly with surface charge density


  10.0 C/m2 rotates with an angular velocity   70 rad/s about the axis passing through its
centre. Find the magnetic induction at the centre of the sphere.

10. A spherical shell of radius a carries a surface charge density  . The shell rotates about the z-axis at
an angular velocity  .
(a) Find the magnetic field at the center of the sphere.
(b) Calculate the magnetic dipole moment of the rotating spherical shell.

11. A square loop of side 6 cm carries a current of 30 A. Calculate the magnitude of magnetic field B at
a point P lying on the axis of the loop and at a distance 7 cm from centre of the loop.

12. A uniform current of the density j flows inside an infinite plate of thickness 2d parallel to its surface.
Find the magnetic induction induced by this current as a function of the distance x from the median
plane of the plate. The magnetic permeability is assumed to be equal to unity both inside and outside
the plate.

13. A long non-magnetic cylindrical conductor with inner radius a and outer radius b carries a current I.
The current density in the conductor is uniform. Assume v
end effects can be neglected
(a) Prove by the Bio-Savart law that the magnetic field.
r b
If any, at a point is always tangential to the circle I
passing through the point, with the centre of the a
circle on the axis of the cylinder. L
(b) Find the magnetic field due to this current as a
function of radius
(i) inside the hollow space (r < a)
(ii) within the conductor (a < r < b)
(iii) outside the conductor (r > b).

14. A very long straight solenoid has a cross-section radius R and n turns per unit length. A direct
current I flows through the solenoid. Supposed that x is the distance from the end of the solenoid,
measured along its axis. Find:
(a) the magnetic induction B on the axis as function of x; draw an approximate plot of B vs the ratio
x/R;
CENTERS : MUMBAI /DELHI /AKOLA /LUCKNOW /NASHIK /PUNE /NAGPUR /BOKARO /DUBAI # 46
MAGNETISM Rg. 2017 - 2019

(b) the distance x0 to the point on the axis at which the value of B differs by   1% from that in the
middle section of the solenoid.

15. A finite conductor AB carrying current i is placed near a fixed very


long current carrying wire i0 as shown in the figure. Find the i
magnitude of the net ampere force on the conductor AB. What i0 A B
a l
happens to the conductor AB if it is free to move. (Neglect
gravitational field)

16. A direct current I flows in long straight conductor whose cross-section has the form of thin half-ring
of radius R. the same current flows in the opposite direction along a thin conductor located on the
“axis” of the first conductor . Find the magnetic interaction force between the given conductors
reduced to a unit of their length.

17. Two long thin parallel conductors of the shape shown in Fig. carry
a b
direct current I1 and I2. The separation between the conductors is a,
the width of the right-hand conductor is equal to b, with both
conductors lying in one plane, find the magnetic interaction force I1
I2
between them reduced to a unit of their length.

18. An infinite wire, placed along z-axis, has current I1 in positive z-direction. A conducting rod placed
parallel to y-axis has current I2 in positive y-direction. The ends of the rod subtend + 30o and -600 at
the origin with positive x-direction. The rod is at the distance a from the origin. Find net force on the
rod. y

19. An equilateral triangular frame with side a carrying a current I is placed at a


x
distance a from an infinitely long straight wire carrying a current I as shown
I 2
in the figure. One side of the frame is parallel to the wire. The whole system I
lies in the x-y plane. Find the magnetic force F acting on the frame. I 3
a I
1

y
g

20. A conductor carries a constant current I along the closed path b f


c
abcdefgha involving 8 of the 12 edges of length l. find the
h
magnetic dipole moment of the closed path. a e
z x
d
21. Find the ratio of magnetic dipole moment to magnetic field at the centre of a disc of radius R which
is rotating at constant angular speed  about its axis. The disc is insulating and uniformly charged.

22. Q charge is uniformly distributed over the slant surface of a right circular cone of semi-vertex angle
 and height h. The cone is uniformly rotated about its axis at angular velocity  . Calculate the
associated magnetic dipole moment.

CENTERS : MUMBAI /DELHI /AKOLA /LUCKNOW /NASHIK /PUNE /NAGPUR /BOKARO /DUBAI # 47
MAGNETISM Rg. 2017 - 2019
z
23. A rectangular coil of size 3.0 cm  4.0 cm and having 100 turns,
is pivoted about the z-axis as shown in figure. The coil carries an
electric current of 2.0 A and a magnetic field of 1.0 T is present
along the y-axis. Find the torque acting on the coil if the side in 2A
the x-y plane makes an angle
 = 37o with the x-axis. y

x
y

24. The figure shows one quarter of a single circular loop of wire. It
carries a current of 14 A. Radius of the loop is a = 5 cm. A uniform
magnetic field: B = 3.0  10 -2 T is directed in +  direction. Find the
torque on the entire loop and the direction on which it will rotate. O
x
60
z
25. Consider a rectangular frame ABCD whose sides are a and b z
meters long. The frame holds N coils, each of which carries a
 D
current I. The frame is embedded in a uniform magnetic field B
along positive x axis which forms an angle  with the frame C
plane.
A y
(a) Find the forces which act on the sides of the frame I
(b) Compute the torque around the DA axis. 
B
x

26. A very small circular loop of radius r and carrying a current i1 is placed in the x-y plane with its
centre on x-axis at the point C(a.0). A square loop of side length 2l carrying a current i2 is fixed in
the y-z plane with the centre of the loop at the origin. Calculate the torque exerted by the square loop
on the circular loop.

27. A square coil of side L = 10 cm is mounted on a fixed insulating axis passing through the midpoints
of a pair of opposite sides. A very long straight conductor parallel to the axis and at a distance of d =
12 cm from it carries a current of 65 A. When the coil is in the position in which its opposite sides
parallel to the axis are equidistant from the long straight conductor, find the torque acting on it if a
current of 78 A passes through it.
a b
28. A loop with current I is in the field of a long straight wire I0
with current I0. The plane of the loop is perpendicular to 2 I
the straight wire. Find the moment of Ampere’s force
acting on this loop.

29. The circular current carrying loop of radius b shown in the figure is mounted rigidly on the axle,
midway between the two supporting cords. In the absence of an
external magnetic field the tensions in the cords are equal and are L
T0 .
1 i 2
(a) What will be the tensions in the cords when the vertical

magnetic field B is present? A B
(b) Repeat if the field is parallel to the axis.

CENTERS : MUMBAI /DELHI /AKOLA /LUCKNOW /NASHIK /PUNE /NAGPUR /BOKARO /DUBAI # 48
MAGNETISM Rg. 2017 - 2019

y
30. Consider a uniform square frame as shown in figure. Each side of the
frame is of length a. The longitudinal mass density of the frame is  . 30
The frame carries an electric current I and is free to move around its
upper side (side 3 in figure).Compute the magnetic field B in the + y a 3
direction, required to hold the frame tilted 30o away from the vertical 2 a
axis.
4 x
30 1

31. A coil of radius R carries a current i1. Another concentric coil of radius r (r << R) carries a current i2.
Planes of two coils are mutually perpendicular and both the coils are free to rotate about common
diameter. Find the maximum kinetic energy of smaller coil when both coils are released. Masses of
coils are M and m respectively.

32. A square frame carrying a current I = 0.90 A is a located in the same plane as a long straight wire
carrying a current I0 = 5.0 A, the frame side has a length a = 8.0 cm. The axis of the frame passing
through the midpoints of opposite sides is parallel to the wire and is separated from it by the distance
which is   1.5 times greater than the sides of the frame. Find:
(a) Ampere force acting on the frame;
(b) the mechanical work to be performed in order to turn the frame through 180o about its axis, with
the currents maintained constant. B
33. A copper wire cross-sectional area S = 2.5 mm2 bent to make three sides of a O
square can turn about a horizontal axis OO as shown in the figure. The wire O
is located in uniform vertical magnetic field. Find the magnetic induction if
on passing a current I = 16 A through the wire the latter deflect by an angle 
  20o.

34. A small current-carrying loop is located at a distance r from a long straight conductor with current I.
The magnetic moment of the loop is equal to pm. Find the magnitude and the direction of the force
vector applied to the loop if the vector p m.
(a) is parallel to the straight conductor;
(b) is oriented along the radius vector r;
(c) coincides in direction with the magnetic field produced by the current I at the point where the
loop is located.

35. Find the interaction force of two coils with magnetic moment P1m = 4.0 mA.m2 and P2m = 6.0
mA.m2, if the separation between the coils is equal to L = 20 cm which exceeds considerably their
linear dimensions.

36. A square current carrying loop made of thin wire and having a mass m = 10 g O B
can rotate without friction with respect to the vertical axis OO’ passing through
the centre of the loop at right angles to two opposite sides of the loop. The loop
is placed in a homogeneous magnetic field with an induction B = 10 -1 T directed I
at right angles to the plane of the drawing. A current I = 2A is flowing in the
loop. Find the period of small oscillations that the loop performs about its
position of stable equilibrium. O

CENTERS : MUMBAI /DELHI /AKOLA /LUCKNOW /NASHIK /PUNE /NAGPUR /BOKARO /DUBAI # 49
MAGNETISM Rg. 2017 - 2019

I
37. A circular loop made up of a stretchable wire carrying current I is placed in a
uniform magnetic field B normal to the plane of loop. Force constant for the
wire is K. Radius of the loop in absence of magnetic field is R. Find the change B
in the radius of loop when placed in the given magnetic field.
 B

38. A charged particle having mass m and charge q is accelerated by a potential difference V, it flies
through a uniform transverse magnetic field B. The field occupies a region of space d. Find the time
interval for which it remains inside the magnetic field.

39. A Beam of equally charged particles after being accelerated through a A


voltage V enters into a magnetic field ‘B’ as shown in the figure. It is found 1 O B 
that all the particles hit plate between C and D. find the ratio between the
C
masses of the heaviest and lightest particles of the beam. d
D

40. A proton of charge e and mass m enters a uniform magnetic field B = Bi with an initial velocity
v  vx i  v y j . Find an expression in unit-vector notation for its velocity at time t.

41. A particle having mass m and charge q is released from the origin in a region in which electric field
and magnetic field are given by B   B0 j and E  E0 k . Find the components of the velocity and the
speed of the particle as a function of its z-co-ordinate.

42. An electron is released from the origin at a place where a uniform electric field E and a uniform
magnetic field B exist along the negative y-axis and the negative z-axis respectively, find the
displacement of the electron along the y-axis when its velocity becomes perpendicular to the electric
field for the first time.

IIT-JEE PROBLEMS

SINGLE CHOICE
1. A particle of charge q and mass m moves in a circular orbit of radius r with angular speed  . The
ratio of the magnitude of its magnetic moment to that of its angular momentum depends on (2000S)
(a)  and q (b)  , q and m (c) q and m (d)  and m

2. Two long parallel wires are at a distance 2d apart. They carry steady equal currents flowing out of
the plane of the paper, as shown. The variation of the magnetic field B along the line XX ' is given
by (2000S)
(a) (b)

(c) (d)

CENTERS : MUMBAI /DELHI /AKOLA /LUCKNOW /NASHIK /PUNE /NAGPUR /BOKARO /DUBAI # 50
MAGNETISM Rg. 2017 - 2019

3. An infinitely long conductor PQR is bent to form a right


angle as shown in Figure. A current I flows through PQR .
The magnetic field due to this current at the point M is
H1 .Now, another infinitely long straight conductor QS is
connected at Q so that current is I 2 in QR as well as in
QS , the current in PQ remaining unchanged. The magnetic
field at M is now H 2 . The ratio H1 H 2 is given by
(2000S)

(a) 1 2 (b) 1 (c) 2 3 (d) 2

4. An ionized gas contains both positive and negative ions. If it is subjected simultaneously to an
electric field along the  x  direction and a magnetic field along the  z direction, then
(a) positive ions deflect towards  y  direction and negative ions towards  y direction
(b) all ions deflect towards  y  direction
(c) all ions deflect towards  y - direction
(d) positive ions deflect towards  y  direction and negative ions towards  y  direction

5. A non-planar loop of conducting wire carrying a current I is placed


as shown in the figure. Each of the straight sections of the loop is of
length 2a . The magnetic field due to this loop at the point P (a,0,a)
points in the direction. (2001 S)
1 1
(a)
2

 ˆj  kˆ (b)
3

 ˆj  kˆ  iˆ 
1 ˆ ˆ ˆ 1 ˆ ˆ
(c)
3

i  jk  (d)
2

i k 
6. Two particles A and B of masses m A and mB respectively and having the same charge are moving
in a plane. A uniform magnetic field exists perpendicular to this plane. The speeds of the particles
are v A and vB respectively and the trajectories are as shown in the figure. Then (2001 S)

(a) m Av A  mB vB (b) m Av A  mB vB
(c) mA  mB and v A  vB (d) mA  mB and v A  vB

7. A coil having N turns is wound tightly in the form of a spiral with inner and outer radii a and
b respectively. When a current I passes through the coil, the magnetic field at the center is (2001 S)
 0 NI 2  0 NI 0 NI b 0 IN a
(a) (b) (c) ln (d) ln
b a 2 b  a  a 2 b  a  b

CENTERS : MUMBAI /DELHI /AKOLA /LUCKNOW /NASHIK /PUNE /NAGPUR /BOKARO /DUBAI # 51
MAGNETISM Rg. 2017 - 2019

8. A particle of mass m and charge q moves with a constant velocity v along the positive
x  direction. It enters a region containing a uniform magnetic field B directed along the negative
z  direction, extending from x  a to x  b . The minimum value of v required so that the particle
can just enter the region x  b is (2002 S)
qbB q b  a B qaB q b  a B
(a) (b) (c) (d)
m m m 2m
9. A long straight wire along the Z-axis carries a current I in the negative Z-direction. The magnetic

field B at a point having coordinates  x, y  in the Z  0 plane is (2002 S)
0 I  yiˆ  xj  0 I  xj  yjˆ   0 I  xjˆ  yjˆ   0 I  xjˆ  yjˆ 
(a) (b) (c) (d)
2  x 2  y 2  2  x 2  y 2  2  x 2  y 2  2  x 2  y 2 

10. The magnetic field line due to a bar magnet are correctly shown in

(a) (b)

(c) (d)

11. For a positively charged particle moving in a x  y plane initially along the x  axis, there is a
sudden change in its path due to the presence of electric and/ or magnetic fields beyond P . The
curved path is shown in the x  y plane and is found to be non-circular. Which one of the following
combinations is possible? ( 2003 S)

   
(a) E  0; B  biˆ  ckˆ (b) E  aiˆ; B  ckˆ  aiˆ
   
(c) E  0; B  cjˆ  bkˆ (d) E  aiˆ; B  ckˆ  bjˆ

CENTERS : MUMBAI /DELHI /AKOLA /LUCKNOW /NASHIK /PUNE /NAGPUR /BOKARO /DUBAI # 52
MAGNETISM Rg. 2017 - 2019

12. A conducting loop carrying a current I is placed in a uniform magnetic field pointing into the plane
of the paper as shown. The loop will have a tendency to (2003 S)

(a) contract (b) expand


(c) move towards  ve x  axis (d) move towards  ve x  axis

13. A current carrying loop is placed in a uniform magnetic field in four different orientations. I,II,III
&IV arrange them in the decreasing order of Potential Energy (2003 S)

(a) I>III>II>IV (b) I>II>III>IV (c) I>IV>II>III (d) III>IV>I>II

14. An electron travelling with a speed u along the positive x  axis enters into a region of magnetic
field where B   B0 kˆ  x  0  . It comes out of the region with speed v then (2004 S)

(a) v  u at y  0 (b) v  u at y  0 (c) v  u at y  0 (d) v  u at y  0


 
15. A magnetic field B  B0 Jˆ , exists in the region a  x  2a , and B   B0 ˆj , in the region
2 a  x  3a , where B0 is a positive constant. A positive point charge moving with a velocity

v  v0iˆ , where v0 is a positive constant, enters the magnetic field at x  a . The trajectory of the
charge in this region can be like
(2007)

CENTERS : MUMBAI /DELHI /AKOLA /LUCKNOW /NASHIK /PUNE /NAGPUR /BOKARO /DUBAI # 53
MAGNETISM Rg. 2017 - 2019

(a) (b)

(c) (d) Z

16. A thin flexible wire of length L is connected to two adjacent


fixed points and carries a current I in the clockwise direction, as
shown in the figure. When the system is put in a uniform
magnetic field of strength B going into the plane of the paper, the
wire takes the shape of a circle. The tension in the wire is
(2010)
IBL IBL IBL
(a) IBL (b) (c) (d)
 2 4

MULTIPLE CHOICE

17. A long current carrying wire, carrying current I1 such that I1 is flowing out from the plane of paper
then for loop ABCD,

(a) the net force is zero


(b) the net torque is zero
(c) as seen from O , the loop will rotate in clockwise along OO ' axis
(d) as seen from O , the loop will rotate in anticlockwise direction along OO ' axis

18. A particle of mass m and charge q , moving with velocity


v enters Region II normal to the boundary as shown in the
figure. Region II has a uniform magnetic field B perpendicular
to the plane of the paper. The length of the Region II is 
Choose the correct choice (s). (2008)

qB
(a) The particle enters Region III only if its velocity v 
m
qB
(b) The particle enters Region III only if its velocity v 
m

CENTERS : MUMBAI /DELHI /AKOLA /LUCKNOW /NASHIK /PUNE /NAGPUR /BOKARO /DUBAI # 54
MAGNETISM Rg. 2017 - 2019

qB
(c) Path length of the particle in Region II is maximum when velocity v 
m
(d) Time spent in Region II is same for any velocity v as long as the particle returns to Region I

SUBJECTIVE PROBLEMS

19. A wheel of radius R having charge Q , uniformly distributed on the rim


of the wheel is free to rotate about a light horizontal rod. The rod is
suspended by light inextensible strings and a magnetic field B is
applied as shown in the figure. The initial tensions in the strings are T0 .
3T0
If the breaking tension of the strings are , find the maximum angular
2
velocity 0 with which wheel can be rotated. (2003-4 Marks)
20. A proton and an   particle are accelerated with same potential
difference and they enter in the region of constant magnetic field B perpendicular to the velocity of
particles. Find the ratio of radius of curvature of proton to the radius of curvature of   particle.
(2004-2 Marks)

21. In a moving coil galvanometer, torque on the coil can be expressed as   ki , where i is current
through the wire and k is constant. The rectangular coil of the galvanometer having number of turns
N , area A and moment of inertia I is placed in magnetic field B . Find
(2005-6 Marks)
(a) k in terms of given parameters N , I , A and B .
(b) the torsion constant of the spring, if a current i0 produces a deflection of  2 in the coil.
(c) the maximum angle through which the coil is deflected, if charge Q is passed through the coil,
almost instantaneously. (ignore the damping in mechanical oscillations).

22. A steady current I goes through a wire loop PQR having shape of a right angle triangle with
PQ  3x, PR  4 x and QR  5 x . If the magnitude of the magnetic field at P due to this loop is
 I 
k  0  , find the value of k . (2009)
 48 x 
23. A rectangular loop PQRS made from a uniform wire has length a. z
width b and mass m. It is free to rotate about the arm PQ, which
remains hinged along a horizontal line taken as the y-axis (see
figure). Take the vertically upward direction as the z-axis. A
uniform magnetic field B = (3i + 4k) B0 exists in the region. The
loop is held in the x-y plane and a current I is passed through it. The P Q
loop is now released and is found to stay in the horizontal position y
in equilibrium.
a
(a) What is the direction of the current I in PQ?
(b) Find the magnetic force on the arm RS. S R
x b
(c) Find the expression for I in terms of B0, a, b and m.

CENTERS : MUMBAI /DELHI /AKOLA /LUCKNOW /NASHIK /PUNE /NAGPUR /BOKARO /DUBAI # 55
MAGNETISM Rg. 2017 - 2019

24. A current of 10 A flows around a closed path in a circuit which is along the
horizontal plane as shown in the figure. The circuit consists of eight alternating arcs D
of radii r1 = 0.08 m and r2 = 0.12 m. Each arc subtends the same angle at the center
r2 C
(a) Find the magnetic field produced by this circuit at the center A
(b) An infinitely long straight wire carrying a current of 10 A is passing through the r1
center of the above circuit placed vertically with the direction of the current being
into the plane of the circuit. What is the force acting on the wire at the center due to
the current in the circuit? What is the force acting on the arc AC and the straight
segment CD due to the current at the center?

25. A long circular tube of length 10 m and radius 0.3 m carries a current along its
curved surface as shown. A wire loop of resistance 0.005 ohm and radius 0.1 m is
placed inside the tube with its axis coinciding with the axis of the tube. The
current varies as I = I0 cos (300 t) where I0 is constant. If the magnetic moment of
the loop is N 0 I 0 sin  300t  , then ‘N’ is (2011)

26. A cylindrical cavity of diameter a exists inside a cylinder of diameter 2a shown in the figure. Both
the cylinder and the cavity are infinitely long. A uniform current density J flows along the length. If
N
the magnitude of the magnetic filed at the point P is given by  0 aJ, then the value of N is:
12

a
P O

2a

27. Two parallel wires in the plane of the paper are distance X0 apart. A point charge is moving with
speed u between the wires in the same plane at a distance X1 from one of the wires. When the wires
carry current of magnitude I in the same direction, the radius of curvature of the path of the point
charge is R1. In contrast, if the currents I in the two wires have directions opposite to each other, the
x R
radius of curvature of the path is R2. If 0  3 , the value of 1 is
x1 R2
COMPREHENSION BASED QUESTIONS

PARAGRAPH – I

Advanced countries are making use of powerful electromagnets to move trains at very high speed. These
trains are called maglev trains (abbreviated from magnetic levitation). These trains float on a guideway and do
not run on steel rail tracks. Instead of using an engine based on fossil fuels, they make use of magnetic field
forces. The magnetized coils are arranged in the guide way which repels the strong magnets placed in the
train’s under carriage. This helps train move over the guideway, a technic called electro-dynamic suspension.
When current passes in the coils of guideway, a typical magnetic field is set up between the undercarriage of
train and guideway which pushes and pull the train along the guideway depending on the requirement. The
lack of friction and its aerodynamic style allows the train to more at very high speed.
CENTERS : MUMBAI /DELHI /AKOLA /LUCKNOW /NASHIK /PUNE /NAGPUR /BOKARO /DUBAI # 56
MAGNETISM Rg. 2017 - 2019

28. The levitation of the train is due to (2006-5M,-2)


(a) Mechanical force (b) Electrostatic attraction
(c) Electrostatic repulsion (d) Magnetic repulsion

29. The disadvantage of maglev trains is that (2006-5M,-2)


(a) More friction (b) Less pollution (c) Less wear & tear (d) High initial cost

30. The force which makes maglev move (2006-5M,-2)


(a) Gravitational field (b) Magnetic field (c) Nuclear forces (d) Air drag

PARAGRAPH – II

The figure shows a circular loop of radius a with two long parallel wires (numbered 1 and 2) all in the plane
of the paper. The distance of each wire form the center of the loop is d. The loop and the wires carrying the
same current I. The current in the loop is in the counterclockwise direction if seen from above.
d
Q S
Wire 1 a
Wire 2

P R

31. When d  a but wires are not touching the loop, it is found that the net magnetic field on the axis of the loop
is zero at a height h above the loop. In that cases (2014)
(a) current in wire 1 and wire 2 is the direction PQ and RS, respectively and h  a
(b) current in wire 1 and wire 2 is the direction PQ and SR, respectively and h  a
(c) current in wire 1 and wire 2 is the direction P Q and SR, respectively and h  1.2a
(d) current in wire 1 and wire 2 is the direction PQ and RS, respectively and h  1.2a

32. Consider d  a , and the loop is rotated about its diameter parallel to the wires by 300 from the position
shown in the figure. If the currents in the wires are in the opposite directions, the torque on the loop at its new
position will be (assume that the due to the wires is constant over the loop) (2014)
 0 I2a 2  0 I2a 2 3  0 I2a 2 3  0 I2a 2
(a) (b) (c) (d)
d 2d d 2d

ASSERTION & REASON

33. STATEMENT-1: The sensitivity of a moving coil galvanometer is increased by placing a suitable magnetic
material as a core inside the coil.

STATEMENT-2: Soft iron has a high magnetic permeability and cannot be easily magnetized or
demagnetized. (2008)
(a) STATEMENT-1 is True, STATEMENT-2 is True; STATEMENT-2 is a correct explanation for
STATEMENT-1.
(b) STATEMENT-1 is True, STATEMENT-2 is True; STATEMENT-2 is NOT a correct explanation for
STATEMENT-1.
(c) STATEMENT-1 is True, STATEMENT-2 is False
(d) STATEMENT-1 is False, STATEMENT-2 is True.

CENTERS : MUMBAI /DELHI /AKOLA /LUCKNOW /NASHIK /PUNE /NAGPUR /BOKARO /DUBAI # 57
MAGNETISM Rg. 2017 - 2019

ANSWER KEY

EXERCISE - 1

Q.1 D Q.2 C Q.3 C Q.4 A Q.5 A Q.6 A Q.7 D

Q.8 A Q.9 A Q.10 B Q.11 B Q.12 C Q.13 A Q.14 A

Q.15 B Q.16 A Q.17 C Q.18 A Q.19 B Q.20 B Q.21 A

Q.22 B Q.23 D Q.24 D Q.25 B Q.26 A Q.27 D Q.28 B

Q.29 C Q.30 C Q.31 C Q.32 A Q.33 C

EXERCISE – 2
1. D 2. A 3. B 4. D 5. A 6. A, B, D 7. B, C, D

8. D 9. A 10. B, D 11. A, B 12. B, C, D 13. B, C 14. C, D

15. A, B 16. A, D 17. B, D 18. A, D 19. A, B 20. A, B, C

EXERCISE – 3

1. B 2. B 3. B 4. B 5. B 6. B 7. B

8. A 9. B 10. A  (r) ; B  (q) ; C  (s) ; D  (p)

11. A  (r) ; B  (s) ; C  (p) ; D  (q) 12.D 13.C

EXERCISE – 4

o I  2     o I  3 2
1.  a  B    b B   
4  a b 4  2 a b 
2. B      tan    o I / 2 R  28T
3. B  4 o I /  d sin   0.10 mT
4.  a  B   0 / 4  4   2 I / R  0.30T  b  B   0 / 4   2  2   2 I / R  0.34T
 c  B   0 / 4  2 I / R  0.11 T
i     I  0 I 2  iˆ 3 ˆj 
5. y   1 x 6. (a)  0    along y-axis (b)   
 i2     a  a 8 8 

7. B  ( 0 / 4 ) I 2 l  2.0T 8. (a) B  ½ 0 R (b) pm  ¼  R 4


2   Q    Q a 2 
9. B  2 3 0 R  29 pT 10. (a) B   0 k , where Q  4 a 2 (b) M  k
3  4 a  3

CENTERS : MUMBAI /DELHI /AKOLA /LUCKNOW /NASHIK /PUNE /NAGPUR /BOKARO /DUBAI # 58
MAGNETISM Rg. 2017 - 2019

  jx inside the plate


11. 2.7  104 T 12. B   0
 0 jd outside the plate
0 I q L
13. (b) B  0 for r  a (c) Deflection 
2 mvr
0 I ( r 2  a 2 ) 0 I
 for a  r  b ,  for r  b
2 r (b2  a 2 ) 2 r
14. (a) B  ½ 0 nI (1  x / x 2  R 2 ) , where x  0 outside and x  0 inside the solenoid
(b) x0  R(1  2 ) / 2  (1   )  5 R

0i i0  l  0 2 I1 I 2
15. F  ln 1   in the direction of i0 16. F1   0 I 2 /  2 R 17. F1  ln(1  b / a )
2  a 4 b

o I1I 2 0 I 2  0 I 2 2  3 
18. ln (3) along negative z-direction. 19. F  i ln j
4 2  2
   R3 Q h 2 tan 2  
20. M  I l 2 j 21. 22. 23. (0.14 k ) N  m
2 0 4
     
24. 0.00072 N-m 25. (a) FAB   FCD   NIaB sin  k , FBC   FDA  NI b B j

(b)   m B cos  k
2 0i1i2l 2 r 2 u0 I I 0 (b  a )sin 
26. 27. 7.2  10 5 N  m 28.
2 2
(a  l ) a  2l 2 2 

 b2iB  b2iB 2 g 0 r 2i1i2 MR


29. (a) T1  T0  T2  T0  (b) T1  T0  T2 30. 31.
L L 3I 2( MR 2  mr 2 )

32. (a) F  2  0 II 0 /  (4 2  1)  0.40  N


(b) A  ( 0 aII0 /  ) ln  (2  1) /(2  1)  0.10  J

33. B  (2  gS / I ) tan   10 mT, where  is the density of copper.

34. (a) F  0 (b) F  ( 0 / 4 ) 2 Ipm / r 2 , F  B ;


2
(c) F  (  0 / 4 ) 2 Ipm / r , F  r

IRB
35. F  3 2  0 p1m p2 m /  l 4  9 nN 36. 0.57 s 37.
2 K
2
m d 1 2V m  d
38. t  Here   sin 1   and R  39. 1  
qB R B q  l 
    Be
40. v   x i  y cos t j  y sin t k , where  
m
2
qB0 2qE0 z  qB0  2 2qE0 z 2Em
41. vx  z , v y  0, vz    z , v 42.
m m  m  m eB 2
CENTERS : MUMBAI /DELHI /AKOLA /LUCKNOW /NASHIK /PUNE /NAGPUR /BOKARO /DUBAI # 59
MAGNETISM Rg. 2017 - 2019

IIT-JEE PROBLMES

1. (c) 2. (b) 3. (c) 4. (c)

5. (d) 6. (b) 7. (c) 8. (b)

9. (a) 10. (d) 11. (b) 12. (b)

13. (c) 14. (b) 15. (a) 16. (c)

17. (a,c) 18. (a,c,d)


DT0 1
19. Wmax  20.
BQr 2 2
2Ni0 AB
21. (a) k  NAB (b)

22. k 7

23. (a) I is clockwise i.e. from P to Q. (b) F = IB0b(3k-4i) (c) I = mg/6B0b

24. (a) 6.54 10 5 weber / m2 (b) Force on AC = 0

25. (6) 26. (5) 27. (3) 28. (d) 29. (d)

30. (b) 31. (c) 32. (b) 33. (c)

CENTERS : MUMBAI /DELHI /AKOLA /LUCKNOW /NASHIK /PUNE /NAGPUR /BOKARO /DUBAI # 60
MAGNETISM Rg. 2017 - 2019

IIT–JEE SYLLABUS

Biot Savart’s law and Ampere’s law; Magnetic field near a current – carrying straight wire, along the axis of
a circular coil and inside a long straight solenoid; Force on a moving charge and on a current – carrying wire
in a uniform magnetic field. Magnetic moment of a current loop; Effect of a uniform magnetic field on a
current loop; Moving coil galvanometer.

CONTENTS

SR. NO. TITLE PAGE NO.



1 THEORY 01 – 33

2 EXERCISE – 1 34 – 38

3 EXERCISE – 2 39 – 41

4 EXERCISE – 3 42 – 44

5 EXERCISE – 4 45 – 50

6 WINDOW TO IIT – JEE 50 – 57

7 ANSWERS KEY 58 – 60

MAGNETISM : Tentative Lecture Flow


(Board Syllabus & Booklet Discussion Included)
Lecture no.1 Definition of B, polestrength, Bar magnets, dipole moment of Bar magnet. End - on
- position, Broad - slide - on position force due to one Bar magnet on the other
Lecture no.2 Earth’s magnetic Field, Dip, inclination, dip circle experiment, tangent
galvanometer, tan - A, tan - B, oscillating magnetometers, dia - para & ferro
magnetism
Lecture no.3 Force on a changed particle due to a magnitude field, hellical paths, pitch, force on
a current carrying wire placed in a magnetic field
Lecture no.4 Torque on a closed current carrying loop due to a magnetic field, concept of m
 q

L 2m
Lecture no.5 Biot - Savart law and its applications force between two parallel current carrying
wires
Lecture no.6 Amperes law, Solenoid, toroid

CENTERS : MUMBAI / DELHI /AKOLA / LUCKNOW / NASHIK / PUNE / NAGPUR / BOKARO / DUBAI

You might also like